ORTHOPEDIC MCQS ONLINE QUESTION BANK H3B

ORTHOPEDIC MCQS ONLINE QUESTION BANK H3B

 

Figure 1 Figure 2

A 10-year-old boy has immediate pain after throwing a baseball. He is unable to elevate his arm due to pain. The child has no medical problems. An anteroposterior radiograph of the shoulder is shown in Figure 1. The most likely diagnosis is:

 

1) Giant cell tumor

3) Unicameral bone cyst

2) Aneursymal bone cyst

5) Osteosarcoma

4) Ewingâs tumor

 

The plain radiograph shows a lytic metaphyseal lesion with symmetric expansion and thinned cortices. There is also a minimally displaced fracture. This is the classic appearance of a unicameral bone cyst.

 

Radiographic features of a unicameral bone cyst: Metaphyseal

 

Purely lytic

 

 

Symmetric expansion (about equal in width to the growth plate) Thinned, but intact, cortices

 

The key histologic features are: Thin fibrous lining

 

Scattered giant cells

 

 

Lipophages or foam-filled histiocytes Correct Answer: Unicameral bone cyst

 

 

 

 

 

Figure 1 Figure 2 Figure 3 Figure 4

A 10-year-old boy has immediate pain after throwing a baseball. He is unable to elevate his arm due to pain. The child has no medical problems. An anteroposterior radiograph of the shoulder is shown in Figure 1. A biopsy is performed and the low- and high-power histologic sections are shown in Figures 2 and 3. The most appropriate treatment would be:

 

1) Preoperative chemotherapy followed by surgical resection

3) Surgical resection and allograft reconstruction

2) Chemotherapy and external beam irradiation

5) Curettage and cement augmentation

4) Aspiration and injection with methyl prednisolone acetate

 

To answer this question, one must make a diagnosis based on the radiograph and histologic sections: The diagnosis is unicameral bone cyst.

The plain radiograph shows a lytic metaphyseal lesion with symmetric expansion and thinned cortices. Also present is a minimally displaced fracture. This is the classic appearance of a unicameral bone cyst.

Radiographic features of a unicameral bone cyst:

 

 

Metaphyseal Purely lytic

 

 

Symmetric expansion (about equal in width to the growth plate) Thinned but intact cortices

Histologic features of a unicameral bone cyst:

 

 

Thin fibrous lining Scattered giant cells

 

Lipophages or foam-filled histiocytes

The treatment of unicameral bone cysts is aspiration of the cyst and injection with methyl prednisolone acetate (150 mg to 200 mg). Many cysts resolve with a single injection, whereas some require more than one aspiration/injection.

Correct Answer: Aspiration and injection with methyl prednisolone acetate

 

 

 

 

 

Figure 1 Figure 2 Figure 3

A 72-year-old woman presents with a 2-week history of increasing hip pain following a minor fall. All of her pain is activity related. Physical examination is normal with an excellent range of motion and no focal tenderness. The plain radiographs and T1- and T2-weighted coronal MRI scans are shown in Figures 1, 2 and 3 respectively. The most likely diagnosis is:

 

1) Metastatic carcinoma

3) Lymphoma

2) Multiple myeloma

5) Avascular necrosis

4) Stress fracture

 

The plain radiographs of the hip are normal with the exception of osteopenia. The clue in the case presentation is the weight bearing pain. Stress fractures may occur following a fall in elderly patients. Physical examination is often completely normal.

The MRI scan shows a linear low-signal area across the subcapital region of the femoral neck. This is specific for a stress fracture. The T2-weighted coronal image shows high-signal corresponding to marrow edema surrounding the fracture line.

 

The treatment of this patient is multiple screw fixation of the femoral neck. Correct Answer: Stress fracture

 

3587. (152) Q9-203:

 

 

 

Figure 1 Figure 2 Figure 3

A 72-year-old woman presents with a 2-week history of increasing hip pain following a minor fall. All of her pain is activity related. Physical examination is normal with an excellent range of motion and no focal tenderness. The plain radiographs and T1- and T2-weighted coronal MRI scans are shown in Figures 1, 2 and 3 respectively. The high-signal areas correspond to:

 

1) Normal marrow signal

3) Cortical bone

2) Marrow with edema

5) Neoplasm

4) Articular cartilage

 

The arrows point to high-signal areas that are normal marrow. T1-weighted coronal images are an excellent method to screen the bone marrow to look for lesions. Pathologic tissue on T1-weighted images usually show a low-signal. The low-signal abnormal areas are then sharply contrasted with the normal marrow that is high signal.

 

Correct Answer: Normal marrow signal

 

 

 

 

 

Slide 1 Slide 2 Slide 3

A 55-year-old man has a three month history of increasing elbow pain. His past medical history is significant for coronary artery disease. He has a 30-pack per year smoking history. Initial radiographs of his elbow were normal. The plain radiograph of his elbow is shown three months later in Slide 1 and a biopsy of the destructive lesion is shown in Slide 2.The most likely diagnosis is:

 

1) Metastatic adenocarcinoma

3) Lymphoma

2) Multiple myeloma

5) Paget disease

4) Osteomyelitis

 

The plain radiograph shows a destructive lesion in the metaphysis of the distal humerus with expansion of the medial cortex. This appearance is characteristic of a bone malignancy. The differential diagnosis includes metastatic bone disease, myeloma, lymphoma, primary mesenchymal tumors (chondrosarcoma and malignant fibrous histiocytoma).

 

 

The histology section shows epithelial cells forming glands in a background of fibroblasts. This patient has metastatic carcinoma. Correct Answer: Metastatic adenocarcinoma

 

 

 

 

Slide 1 Slide 2 Slide 3

A 50-year-old man has a 3-month history of increasing pain in his arm. He is a plumber and the pain has increased in his arm to the point where he cannot work. He is otherwise healthy and has no medical problems. His plain radiograph (Slide 1) and magnetic resonance image (Slide 2) are shown below. A needle biopsy is also shown (Slide 3).

Other tests are as follows:

 

 

CXR: normal Lab tests:

 

CBC â normal

 

 

CHEM GROUP â normal ESR â normal

The most likely diagnosis based on the history, imaging studies, and biopsy is:

 

1) Metastatic bone disease

3) Myeloma

2) Lymphoma

5) Paget disease

4) Hyperparathyroidism

 

The correct diagnosis is myeloma of bone. The radiographic appearance of this lesion reveals a pathologic fracture and a diffuse lytic pattern. These can be seen in metastatic bone disease, myeloma, lymphoma, and hyperparathyroidism. In this particular question, the examinee must be able to recognize the histologic features of the myeloma â plasma cell neoplasm.

 

The essential features of plasma cells are: Eccentric nucleus

 

Peripheral chromatin pattern

 

A perinuclear halo (site of the Golgi apparatus)

Myeloma is a common disease and all physicians should be aware of its essential features.Correct Answer: Myeloma

 

 

 

 

 

Slide 1 Slide 2 Slide 3 Slide 4

A 50-year-old woman presented with a 1-year history of shin and knee pain. A presumptive diagnosis of Paget disease had been made and she was treated with anti-Pagetic medications. Her pain continued and a new anteroposterior radiograph of her femur was taken (Slide 1). A coronal and axial magnetic resonance imaging scan (Slides 2 and Slide 3) and a biopsy of the soft tissue mass were also taken (Slide 4).Immunostaining was negative for keratin but postive for CD20

The most likely diagnosis is:

 

1) Metastatic bone disease

3) Lymphoma

2) Myeloma of bone

5) Sarcoma complicating Paget disease

4) Osteosarcoma of bone

 

 

 

 

 

 

 

 

 

 

The histology of lymphoma is a mixed blue cell infiltrate of round cells, which may be small and large.Correct Answer: Lymphoma 3591. (941) Q9-1232:

 

Slide 1 Slide 2 Slide 3 Slide 4

An 18-year-old male presented with a 6-month history of hip pain. His pain occurred both at rest and at night. His parents noticed a significant limp, and he felt his leg was much weaker. The plain radiograph and computerized tomography (CT) scan of the hip are shown (Slide 1 and Slide 2). The low- and high-power hematoxylin and eosin histologic sections are also presented (Slide 3 and Slide 4).

The most likely diagnosis is:

 

1) Low-grade osteosarcoma

3) Parosteal osteosarcoma

2) High-grade intramedullary osteosarcoma

5) Osteoid osteoma

4) Osteoblastoma

 

Osteoid osteoma is a self-limited benign bone tumor with a small nidus (usually less than 1.5 cm) surrounded by reactive bone formation. Histologically, the lesion is well demarcated.The CT scan shows the lucent nidus is surrounded by scelentic bone.Correct Answer: Osteoid osteoma

 

 

 

 

 

Slide 1 Slide 2 Slide 3

A 13-year-old boy presented with a 6-month history of shoulder pain. His pain occurs both at rest and with activity. He feels the pain at night and has difficulty sleeping. The plain radiograph is shown (Slide 1). The low- and high-power hematoxylin and eosin sections are also presented (Slide 2 and Slide 3).

The most likely diagnosis based on the radiographs and biopsy material is:

 

1) Osteoblastoma

3) Parosteal osteosarcoma

2) High-grade intramedullary osteosarcoma

5) Osteomyelitis

4) Osteoid osteoma

 

Osteoblastomas typically have an oval area of bone destruction and there is evidence of bone production within the lesion. Histologically, one can see anastomosing bony trabeculae in a loose fibrovascular stroma. The bone trabeculae are usually lined with osteoblasts.Correct Answer: Osteoblastoma

 

 

3593. (943) Q9-1234:

 

 

 

Slide 1 Slide 2 Slide 3 Slide 4

A 13-year-old boy has a 4-month history of increasing knee pain. A plain radiograph of the knee is shown (Slide 1) and a technetium bone scan (Slide 2). The low- and high-power hematoxylin/eosin sections of the biopsy are presented (Slide 3 and Slide 4).

The most likely diagnosis based on the age of the patient, location of the tumor, and the radiographic images with the biopsy material is:

 

1) Osteomyelitis

3) High-grade intramedullary osteosarcoma

2) Eosinophilic granuloma

5) Nonossifying fibroma

4) Ewing tumor

 

The radiograph shows the classic appearance of osteogenic sarcoma in adolescents: metaphyseal location, bone destruction and bone formation, and a mineralized soft tissue mass. The histologic sections show a malignant stroma of spindle cells producing osteoid.Correct Answer: High-grade intramedullary osteosarcoma

 

 

 

 

 

Slide 1 Slide 2 Slide 3 Slide 4

 

 

 

Slide 5

A 30-year-old woman has a painless mass in the back of her knee. She has felt the mass for over six months. She has restricted range of motion in the knee and is now having difficulty when she walks. The plain radiographs are shown (Slide 1 and Slide 2). The low- and high-power histologic hematoxylin/eosin sections are also presented (Slide 3, Slide 4 and Slide 5).

Based upon the history, physical examination, plain radiographs, and biopsy the most likely diagnosis is:

 

1) Heterotopic ossification

3) High-grade intramedullary osteosarcoma

2) Osteochondroma

5) Periosteal osteosarcoma

4) Parosteal osteosarcoma

 

The plain radiographs show a heavily mineralized lesion on the surface of the bone. The mass encircles the bone. Note the prominent bone formation. The low-power histologic section shows prominent bone formation and fibrous components. The high-power section shows a spindle cell tumor with prominent fibrous tissue production. The spindle cells are uniform in appearance . The nuclei lack pleomorphism. This is a parosteal osteosarcoma.Correct Answer: Parosteal osteosarcoma

 

 

3595. (945) Q9-1236:

 

 

 

Slide 1

A 40-year-old man injures his knee sliding into second base while playing baseball. The radiograph of the knee shows a lesion in the distal femur (Slide 1). Prior to the accident, he had no pain and is otherwise healthy. He is now asymptomatic.

The most likely diagnosis based on the plain radiograph is:

 

1) Dedifferentiated chondrosarcoma

3) Osteochondroma

2) Low-grade intramedullary chondrosarcoma

5) Enchondroma

4) Chondroblastoma

 

Enchondromas are cartilage tumors that are usually inactive in adults. They usually do not grow or cause any symptoms. They are intramedullary lesions and one can recognize their characteristic matrix mineralization: stipples and rings.Correct Answer: Enchondroma

 

 

 

 

 

Slide 1 Slide 2 Slide 3

A 20-year-old man bangs his shoulder while playing ice hockey one week ago, and now he has pain and a mass over the deltoid area. A plain radiograph is shown (Slide 1). An open biopsy is performed and the low- and high-power sections are presented (Slide 2 and Slide 3).

The most likely diagnosis based on the radiographic features and biopsy is:

 

1) Osteochondroma

3) Heterotopic ossification

2) Low-grade intramedullary chondrosarcoma

5) Periosteal chondroma

4) Parosteal osteosarcoma

 

Periosteal chondroma is an uncommon surface cartilage tumor. The lesion is usually small (3 cm to 5 cm) and rests in a saucer-shaped depression on the surface of the bone. Characteristic locations include the proximal humerus and distal femur.

Histologically, the lesion is composed of lobules of cartilage. Although this tumor is benign, the lesion is usually hyper cellular and there may be prominent atypia of the cartilage cells. The treatment is excion with a marginal margin.Correct Answer: Periosteal chondroma

 

 

3597. (947) Q9-1238:

 

 

 

Slide 1

A 16-year-old girl has a painless bump behind her knee. However, she does have pain if she her leg is hit while playing soccer. On physical examination there is a firm lump behind the knee. A plain radiograph of the knee is shown (Slide 1).

The most likely diagnosis based on the history, physical examination and plain radiograph is:

 

1) High-grade intramedullary osteosarcoma

3) Periosteal osteosarcoma

2) Parosteal osteosarcoma

5) Periosteal chondroma

4) Osteochondroma

 

Osteochondromas have a characteristic radiographic appearance. The lesion is a surface tumor and the medullary cavity of the normal bone flows into the osteochondroma. Some authors call this "cortical sharing." Osteochondromas may have a well defined stalk (pedunculated osteochondroma) or they may have a broad base as in this case (sessile osteochondroma). This diagnosis can be made with the plain radiographs without histologic confirmation.Correct Answer: Osteochondroma

 

Paget disease is a primary disorder of:

 

1) Epiphyseal plate

3) Synovial membrane

2) Articular cartilage

5) Intravertebral disc

4) Bone

 

Paget disease is a primary disorder of bone. Paget disease is a remodeling disease of bone. There is abnormal increased activity of the osteoclast. The osteoclast resorbs the bone resulting in lysis and the osteoblast lays down new bone in thickened trabeculae and cortices. The treatment of Paget's disease of bone is stopping the osteoclasts. Anti-pagetic medications include the diphosphonate class of medications.Correct Answer: Bone

 

 

3599. (949) Q9-1240:

 

 

 

Slide 1 Slide 2 Slide 3 Slide 4

A 30-year-old attorney has a 6-month history of increasing knee pain. His pain occurs both with activity and at rest. He is otherwise healthy. Laboratory studies show a normal complete blood count, sedimentation rate, and chemistry values (including normal calcium and phosphorus levels). Anteroposterior and lateral radiographs are shown (Slide 1 and Slide 2). The low- and high-power hematoxylin and eosin sections are shown (Slide 3 and Slide 4).

The diagnosis, based on the history, laboratory values, radiographs, and biopsy sections, most likely is:

 

1) Telangiectatic osteosarcoma

3) Hyperparathyroidism

2) Osteomyelitis

5) Aneurysmal bone cyst

4) Giant cell tumor

 

The radiograph shows purely lytic bone destruction that fills the lateral metaphysis and extends to the articular surface. The lateral cortex is very thin and almost completely destroyed. The histologic sections show collection of giant cells and mononuclear cells.

Notice that the giant cells are uniformly scattered throughout the histologic section on the low power, in contrast to chondroblastoma where the giant cells are scattered throughout the mononuclear cells and that the nuclei of the mononuclear cells are the same as the nuclei of the giant cells.Correct Answer: Giant cell tumor

 

 

 

 

 

Slide 1 Slide 2 Slide 3

A 60-year-old man presents with a 6-month history of increasing knee and distal thigh pain. He is otherwise healthy and has no history of prior malignancies. He has discomfort at night and at rest. The plain radiograph of the lesion is shown (Slide 1), and low and high power histologic sections from an open biopsy are shown (Slide 2 and Slide 3).

The most likely diagnosis based on the history, plain radiographs, and biopsy material is:

 

1) Bone infarct

3) Lymphoma

2) Enchondroma

5) Metastatic bone disease

4) Dedifferentiated chondrosarcoma

 

Dedifferentiated chondrosarcomas are high-grade cartilage tumors that are bimorphic (two distinct histologic components). Alongside the very low-grade cartilage tumor are areas of high-grade spindle cell sarcoma. The cartilage component is very low grade (grade ½, grade 1, or grade 2), while the high-grade component is an anaplastic spindle cell sarcoma; either osteosarcoma, fibrosarcoma, or malignant fibrous histiocytoma. The radiographs also reflect the histologic bimorphic component; area typical for low-grade chondrosarcoma with superimposed aggressive area. The aggressive component is usually represented by an area of:

 

 

Lytic bone destruction

 

 

Cortical bone erosion or typically destruction Soft tissue mass

 

 

 

Correct Answer: Dedifferentiated chondrosarcoma 3601. (951) Q9-1244:

 

Slide 1

A 17-year-old boy hurt his knee when he was tackled during a football game. He has a large effusion and experiences difficulty in walking. A plain radiograph of the knee is shown (Slide 1).

The most likely diagnosis based on the plain radiograph is:

 

1) High-grade intramedullary osteosarcoma

3) Chondroblastoma

2) Giant cell tumor

5) Fibrous cortical defect

4) Ewing tumor

 

Fibrous cortical defects are cortically based, small metaphyseal lesions. They have a sclerotic rim with a lucent center. The lesions do not cause symptoms, and no treatment is necessary.Correct Answer: Fibrous cortical defect

 

 

 

 

 

Slide 1 Slide 2

A 55-year-old man presented with a 4-month history of severe knee pain. The plain anteroposterior radiograph of the knee is shown (Slide 1). He has pain both at rest and at night. A staging evaluation was performed to determine of this was a metastatic lesion. A chest radiograph, chest computed tomography (CT) scan, and an abdominal CT scan were all negative. Serum studies showed a normal complete blood cell count, differential, chemistry group, and serum electrophoresis. A biopsy was performed and is shown (Slide 2).

The most likely diagnosis is:

 

1) Metastatic bone disease

3) Lymphoma

2) Multiple myeloma

5) Malignant fibrous histiocytoma

4) Fibrosarcoma

 

The radiographics show a destructive lesion with a mottled appearance: a mixture of bone destruction and bone formation. The biopsy specimen shows the characteristic features of malignant fibrous histiocytoma: a storiform pattern, histiocytes, and bizarre giant cells.Correct Answer: Malignant fibrous histiocytoma

 

 

 

 

 

Slide 1 Slide 2 Slide 3 Slide 4

 

 

 

Slide 5

A 30-year-old man presents with an 18-month history of pain in his leg. The pain is over the mid-section of the tibia and occurs both at rest and activity. There no history of trauma or infections. The plain radiographs are shown (Slide 1 and Slide2) and a biopsy of the lesion is shown (Slide 3, Slide 4 and Slide 5).

The most likely diagnosis based on the clinical findings, plain radiographs, and biopsy specimens is:

 

1) Osteomyelitis

3) Fibrous dysplasia

2) High-grade intramedullary osteosarcoma

5) Adamantinoma

4) Osteofibrous dysplasia

 

Adamantinoma most commonly occurs in the tibia and has multiple areas of bone destruction. There is often intervening bone sclerosis between the areas of bone destruction. The biopsy specimens shows the typical features of adamantinoma: epithelial cells in fibrous stroma.Correct Answer: Adamantinoma

 

 

3604. (954) Q9-1248:

 

 

 

Slide 1 Slide 2 Slide 3 Slide 4

A 55-year-old man presented with a 12-month history of increasing low back pain. He experiences pain during the night and often has difficulty sitting. He has also noticed that in the last 9 months constipation has become an increasing problem. The plain radiographs and computerized tomography (CT) scan are shown (Slide 1 and Slide 2). Low- and high-power histologic sections are also presented (Slide 3 and Slide 4).

The most likely diagnosis is:

 

1) Metastatic adenocarcinoma

3) Malignant fibrous histiocytoma

2) Multiple myeloma

5) Sacral insufficiency fracture

4) Chordoma

 

The classic midline bone destruction with an anterior soft tissue mass is present. The histology shows a lobular pattern of growth and the characteristic physaliferous cells can be seen on high power.Correct Answer: Chordoma

 

 

 

 

 

Slide 1 Slide 2 Slide 3

A 25-year-old man has a 4-month history of severe thigh and knee pain. He has pain both at rest and at night. He has tenderness on palpation of the thigh. He has been febrile and has an elevated white blood cell count. The anteroposterior and lateral radiographs of the femur are shown (Slide1). The high and lower power histologic sections are also shown (Slide 2) and (Slide 3).

The most likely diagnosis is:

 

1) Osteogenic sarcoma

3) Malignant fibrous histiocytoma

2) Osteomyelitis

5) Ewing sarcoma

4) Fibrosarcoma

 

The histologic features of Ewing tumor include a monotonous blue cell tumor, indistinct cell borders, and no matrix production by the tumor cells.Correct Answer: Ewing sarcoma

 

 

3606. (1071) Q9-1429:

 

 

 

Slide 1 Slide 2 Slide 3

A 50-year-old man has a lesion in the left humerus that is discovered during a routine annual physical examination. The plain radiographs (Slide 1) and the coronal T1- and T2-weighted images (Slides 2 and 3) are presented. The most likely diagnosis based on the plain radiographs and magnetic resonance imaging scan is:

 

1) Low grade chondrosarcoma

3) Pagetâs disease

2) Enchondroma

5) Multiple myeloma

4) Metastatic carcinoma

 

The plain radiographs show a mineralized lesion in the medullary cavity. The mineralization shows stipples and small rings. This pattern is consistent with a cartilage lesion. There are no cortical changes indicating that the lesion is inactive. The magnetic resonance imaging scan shows a lesion with a low signal on the T1-weighted image and high signal on the T2-weighted image. There is no marrow edema bordering the lesion on the T2-weighted image. All of these features are compatible with the diagnosis of an enchondroma. Chondrosarcomas in long bones have easily recognized features: large cortical erosions, cortical destruction, cortical thickening, and a soft tissue mass.

 

The other possible answers cover lesions with characteristic radiographic features: Pagetâs disease: Coarsened trabeculae and thickened cortices

 

Metastatic carcinoma: Lytic lesion with cortical bone destruction

 

 

Multiple myeloma: Purely lytic punched-out lesion that may be in a single or multiple areas Correct Answer: Enchondroma

 

 

 

 

Slide 1 Slide 2 Slide 3 Slide 4

A 50-year-old man has a lesion in the left humerus that is discovered during a routine annual physical examination. The plain radiographs (Slide 1), the coronal T1- and T2-weighted images (Slides 2 and 3), and a biopsy (Slide 4) are presented. The most appropriate treatment method is:

 

1) Observation with repeat radiograph in 3 to 6 months

3) Systemic chemotherapy and external beam radiation to the humerus

2) Wide surgical resection and custom prosthetic reconstruction

5) Curettage, rod fixation, and external beam irradiation

4) External beam irradiation alone to the humerus

 

The plain radiographs show a mineralized lesion in the medullary cavity. The mineralization shows stipples and small rings. This pattern is consistent with a cartilage lesion. There are no cortical changes indicating that the lesion is inactive. The magnetic resonance imaging scan shows a lesion with a low signal on the T1-weighted image and high signal on the T2-weighted image. There is no marrow edema bordering the lesion on the T2-weighted image. All of these features are compatible with the diagnosis of an enchondroma. Chondrosarcomas in long bones have easily recognized features: large cortical erosions, cortical destruction, cortical thickening, and a soft tissue mass.

 

The biopsy shows a cartilage lesion. There is no permeation of the trabecular bone. Both the plain radiographs and biopsy specimen are consistent with the diagnosis of an enchondroma.

 

Enchondromas are treated with observation and repeat radiographs in 3 to 6 months. Correct Answer: Observation with repeat radiograph in 3 to 6 months

 

 

 

Slide 1 Slide 2 Slide 3 Slide 4

A 50-year-old man has a lesion in the left humerus that is discovered during a routine annual physical examination. The plain radiographs (Slide 1), the coronal T1- and T2-weighted images (Slides 2 and 3), and a biopsy (Slide 4) are presented. The correct stage of this lesion according to the Musculoskeletal Tumor Society is:

 

1) Stage I

3) Stage III

2) Stage II

5) Stage 2

4) Stage 1

 

The plain radiographs show a mineralized lesion in the medullary cavity. The mineralization shows stipples and small rings. This pattern is consistent with a cartilage lesion. There are no cortical changes indicating that the lesion is inactive. The magnetic resonance imaging scan shows a lesion with a low signal on the T1-weighted image and high signal on the T2-weighted image. There is no marrow edema bordering the lesion on the T2-weighted image. All of these features are compatible with the diagnosis of an enchondroma. Chondrosarcomas in long bones have easily recognized features: large cortical erosions, cortical destruction, cortical thickening, and a soft tissue mass.

 

Enchondromas are almost always inactive and asymptomatic. Therefore, enchondromas are Stage 1 inactive lesions. Remember:

Stage 1 Latent, inactive Stage 2 Active

Stage 3 Aggressive

Stage I Low grade Stage II High grade

 

Stage III Metastatic disease Correct Answer: Stage 1

 

Slide 1 Slide 2 Slide 3

A 50-year-old man has a lesion in the left humerus that was discovered during a routine annual physical examination. The plain radiographs (Slide 1) and the coronal T1- and T2-weighted images (Slides 2 and 3) are presented. The area of mineralization in the medullary cavity marked by the arrow in Figure 1 is most likely:

 

1) Normal marrow which has calcified

3) Areas of metastatic carcinoma

2) Hyaline cartilage which has calcified

5) Fibrous replacement of the marrow

4) Reactive bone formation

 

The plain radiographs show a mineralized lesion in the medullary cavity. The mineralization shows stipples and small rings. This pattern is consistent with a cartilage lesion. There are no cortical changes indicating that this lesion is active. The magnetic resonance imaging scan shows a lesion with a low signal on the T1-weighted image and a high signal on the T2-weighted image. The high signal on the T2-weighted image is reflective of the high water content of cartilage lesions. There is no marrow edema bordering the lesion on the T2-weighted image. All of these features are consistent with the diagnosis of an enchondroma.

 

The other possible answers are wrong for a variety of reasons: Normal marrow does not calcify.

 

Metastatic carcinoma will show lytic features.

 

 

Reactive bone formation shows cloud-like mineralization, and the bone is low signal on T1- and T2-weighted images. When the marrow is replaced by fibrous tissue, there will not be any mineralization.

Correct Answer: Hyaline cartilage which has calcified

 

 

Slide 1 Slide 2

A 29-year-old woman has a 5-month history of dull, aching pain in her shoulder after falling from her bicycle. Her radiograph (Slide 1) and biopsy specimen (Slide 2) are presented. The most likely diagnosis is:

 

1) Telangiectatic osteosarcoma

3) Giant cell tumor

2) Aneurysmal bone cyst

5) Ewing tumor

4) Unicameral bone cyst

 

The plain radiographs show a destructive lytic lesion. The differential diagnosis, based on the patientâs age and the pattern of bone destruction, is:

 

 

Giant cell tumor Aneurysml bone cyst

 

 

Telangiectatic osteosarcoma Ewing tumor

The histology clearly shows a giant cell tumor with uniformly distributed giant cells in a field of mononuclear cells. The diagnosis is giant cell tumor.

Correct Answer: Giant cell tumor

 

 

Slide 1 Slide 2

A 29-year-old woman has a 5-month history of dull, aching pain in her shoulder after falling from her bicycle. Her radiograph (Slide 1) and biopsy specimen (Slide 2) are presented. Which of the following is the most appropriate treatment regimen:

 

1) Preoperative multiagent chemotherapy followed by wide resection

3) Wide resection followed by external beam irradiation

2) Internal fixation and external beam irradiation

5) Forequarter amputation

4) Marginal resection followed by joint reconstruction

 

The plain radiographs show a destructive lytic lesion. The differential diagnosis, based on the patientâs age and the pattern of bone destruction, is:

 

 

Giant cell tumor Aneurysml bone cyst

 

 

Telangiectatic osteosarcoma Ewing tumor

The histology clearly shows a giant cell tumor with uniformly distributed giant cells in a field of mononuclear cells. The diagnosis is giant cell tumor.

The treatment of giant cell tumor is curettage with methylmethacrylate augmentation for lesions in which the joint surfaces can be saved. If the joint surfaces cannot be saved, marginal resection and joint reconstruction must then be performed. Joint reconstruction in this case may be either with an osteoarticular allograft, allograft prosthetic composite, or a metal prosthesis.

 

Correct Answer: Marginal resection followed by joint reconstruction

 

 

Slide 1 Slide 2

A 29-year-old woman has a 5-month history of dull, aching pain in her shoulder after falling from her bicycle. Her radiograph (Slide 1) and biopsy specimen (Slide 2) are presented. The stage of this lesion according to the Musculoskletal Tumor Society would be:

 

1) Stage 1

3) Stage 3

2) Stage 2

5) Stage II

4) Stage I

 

The plain radiographs show a destructive lytic lesion. The differential diagnosis, based on the patientâs age and the pattern of bone destruction, is:

 

 

Giant cell tumor Aneurysml bone cyst

 

 

Telangiectatic osteosarcoma Ewing tumor

The histology clearly shows a giant cell tumor with uniformly distributed giant cells in a field of mononuclear cells. The diagnosis is giant cell tumor.

This tumor is aggressive and has destroyed the entire metaphysis, resulting in a fracture. The epiphysis is also completely destroyed. This is an aggressive lesion and would be classified as a Stage 3 aggressive lesion.

Remember that Arabic numerals are for benign lesions and Roman numerals are for malignant lesions. Remember:

 

Stage 1 Inactive/latent

 

Stage 2 Active

 

Stage 3 Aggressive

 

 

 

Stage I Low grade Stage II High grade

 

 

Stage III Metastatic disease Correct Answer: Stage 3

 

Slide 1 Slide 2 Slide 3 Slide 4

 

 

 

Slide 5 Slide 6

A 50-year-old woman has had severe hip pain for 4 months. Her plain radiographs (Slide 1), technetium bone scan (Slide 2), computerized tomography scan (Slide 3), and coronal T1- and T2-weighted magnetic resonance images (Slide 4) are presented. A needle biopsy is also performed (Slides 5 and 6). The most likely diagnosis is:

 

1) Metastatic adenocarcinoma

3) Lymphoma

2) Multiple myeloma

5) Malignant fibrous histiocytoma

4) Chondrosarcoma

 

The plain radiographs show a purely lytic destructive lesion that is poorly marginated. The technetium bone scan does not show any major uptake. The computerized tomography scan shows purely lytic bone destruction with breakthrough of the cortical bone. Complete destruction of the cortical bone is suggestive of a malignancy. The magnetic resonance image shows a lesion that is homogenously low on T1-weighted images and high on T2-weighted images. Surgeons cannot make a definitive diagnosis based upon the radiographic features. The most common malignancies in this age group are:

 

  1. Metastatic bone disease

  2. Multiple myeloma

  3. Lymphoma

  4. Chondrosarcoma

  5. Malignant fibrous histiocytoma

The biopsy specimen shows round, epithelial-like cells that are grouped in clusters. The cells are arranged in a fibrous background. The diagnosis is metastatic bone disease.

Correct Answer: Metastatic adenocarcinoma

 

 

Slide 1 Slide 2 Slide 3 Slide 4

 

 

 

Slide 5 Slide 6

A 50-year-old woman has had severe hip pain for 4 months. Her plain radiographs (Slide 1), technetium bone scan (Slide 2), computerized tomography scan (Slide 3), and coronal T1- and T2-weighted magnetic resonance images (Slide 4) are presented. A needle biopsy is also performed (Slides 5 and 6). Which of the following treatment regimens would be the most appropriate:

 

1) Wide resection and allograft acetabular reconstruction

3) Preoperative chemotherapy followed by wide resection

2) Internal hemipelvectomy alone

5) Hemipelvectomy

4) Acetabular reconstruction followed by external beam irradiation

 

The plain radiographs show a purely lytic destructive lesion that is poorly marginated. The technetium bone scan does not show any major uptake. The computerized tomography scan shows purely lytic bone destruction with breakthrough of the cortical bone. Complete destruction of the cortical bone is suggestive of a malignancy. The magnetic resonance image shows a lesion that is homogenously low on T1-weighted images and high on T2-weighted images. Surgeons cannot make a definitive diagnosis based upon the radiographic features. The most common malignancies in this age group are:

 

  1. Metastatic bone disease

  2. Multiple myeloma

  3. Lymphoma

  4. Chondrosarcoma

  5. Malignant fibrous histiocytoma

The biopsy specimen shows round, epithelial-like cells that are grouped in clusters. The cells are arranged in a fibrous background. The diagnosis is metastatic bone disease.

The treatment of metatastic bone disease is either internal fixation with postoperative external beam irradiation or external beam irradiation alone. In this case, there is extensive destruction and fracture is imminent. The treatment of choice is acetabular reconstruction followed by external beam irradiation.

 

Correct Answer: Acetabular reconstruction followed by external beam irradiation

 

 

Slide 1 Slide 2 Slide 3 Slide 4

A 50-year-old woman has had severe hip pain for 4 months. Her plain radiographs (Slide 1), technetium bone scan (Slide 2), computerized tomography scan (Slide 3), and coronal T1- and T2-weighted magnetic resonance images (Slide 4) are presented. The most likely diagnosis based upon the radiographs would be:

 

1) Septic arthritis

3) Stress fracture

2) Osteomyelitis of the acetabulum

5) Malignant neoplasm

4) Lytic phase of Pagetâs disease

 

The plain radiographs show a purely lytic destructive lesion that is poorly marginated. The technetium bone scan does not show any major uptake. The computerized tomography scan shows purely lytic bone destruction with breakthrough of the cortical bone. Complete destruction of the cortical bone is suggestive of a malignancy. The magnetic resonance image shows a lesion that is homogenously low on T1-weighted images and high on T2-weighted images. Surgeons cannot make a definitive diagnosis based upon the radiographic features. The most common malignancies in this age group are:

 

  1. Metastatic bone disease

  2. Multiple myeloma

  3. Lymphoma

  4. Chondrosarcoma

     

  5. Malignant fibrous histiocytoma Correct Answer: Malignant neoplasm

 

Slide 1 Slide 2 Slide 3 Slide 4

 

 

 

Slide 5 Slide 6

A 50-year-old woman has had severe hip pain for 4 months. Her plain radiographs (Slide 1), technetium bone scan (Slide 2), computerized tomography scan (Slide 3), and coronal T1- and T2-weighted magnetic resonance images (Slide 4) are presented. A needle biopsy is also performed (Slides 5 and 6). The cells directly responsible for the bone destruction are:

 

1) Tumor cells

3) Fibroblasts

2) Plasma cells

5) Mast cells

4) Osteoclasts

 

The plain radiographs show a purely lytic destructive lesion that is poorly marginated. The technetium bone scan does not show any major uptake. The computerized tomography scan shows purely lytic bone destruction with breakthrough of the cortical bone. Complete destruction of the cortical bone is suggestive of a malignancy. The magnetic resonance image shows a lesion that is homogenously low on T1-weighted images and high on T2-weighted images. Surgeons cannot make a definitive diagnosis based upon the radiographic features; however, it is likely that this lesion is malignant. The biopsy specimen shows round, epithelial-like cells that are grouped in clusters. The cells are arranged in a fibrous background. The diagnosis is metastatic bone disease.

 

The bone destruction in metastatic bone disease is a direct result of osteoclastic resorption of the bone. The metastatic breast cells secrete a factor that activates and recruits osteoclasts that destroy the bone. Diphosphonate therapy (usually intravenous pamidronate) is given to virtually all women with metatstatic breast carcinoma to stop the osteoclasts from resorbing the bone matrix.

 

Correct Answer: Osteoclasts

 

 

Slide 1 Slide 2 Slide 3 Slide 4

A 65-year-old woman has a 5-month history of increasing hip and thigh pain. Her plain radiographs (Slide 1) and T1- and T2-weighted coronal images (Slides 2 and 3) of her hip are presented. A biopsy is also performed (Slide 4). The most likely diagnosis is:

 

1) Multiple myeloma

3) Metastatic adenocarcinoma

2) Lymphoma

5) Pagetâs disease

4) Chondrosarcoma

 

The plain radiographs show an ill-defined permeative lesion in the proximal femur. The lateral cortex of the greater trochanter is thinned. The T1-weighted coronal images show a low-signal lesion in the entire greater trochanter extending to the base of the femoral neck. In the opposite proximal femur and acetabulum, there are multiple small low-signal lesions consistent with metastatic bone disease. The T2-weighted images show a very high-signal area in the greater trochanter. The inversion recovery image is sensitive to pathologic processes. Normal marrow will be low signal and pathologic lesions are high signal. The biopsy shows round epithelial cells forming glands. This pattern is characteristic of metastatic adenocarcinoma.

 

Correct Answer: Metastatic adenocarcinoma

 

 

Slide 1 Slide 2 Slide 3 Slide 4

A 65-year-old woman has a 5-month history of increasing hip and thigh pain. She has severe pain with weight bearing on the right leg. Her plain radiographs (Slide 1) and T1- and T2-weighted coronal images (Slides 2 and 3) of her hip are presented. A biopsy is also performed (Slide 4). The most appropriate treatment would be:

 

1) Hip disarticulation

3) Wide resection and custom proximal femoral replacement

2) Neck exploration and removal of her parathyroid glands

5) Internal fixation followed by external beam irradiation

4) Crutch ambulation for 6 to 8 weeks

 

The plain radiographs show an ill-defined permeative lesion in the proximal femur. The lateral cortex of the greater trochanter is thinned. The T1-weighted coronal images show a low-signal lesion in the entire greater trochanter extending to the base of the femoral neck. In the opposite proximal femur and acetabulum, there are multiple small low-signal lesions consistent with metastatic bone disease. The T2-weighted images show a very high-signal area in the greater trochanter. The inversion recovery image is sensitive to pathologic processes. Normal marrow will be low signal and pathologic lesions are high signal. The biopsy shows round epithelial cells forming glands. This pattern is characteristic of metastatic adenocarcinoma.

 

The treatment of metastatic breast carcinoma of bone is external beam irradiation to symptomatic lesions and diphosphonate therapy to retard the bone loss. Internal fixation is performed to prevent pathologic fracture. When patients have significant weight bearing pain and a purely lytic lesion in the hip, the risk of fracture is high. The best treatment method for this patient is internal fixation followed by external beam irradiation.

 

Correct Answer: Internal fixation followed by external beam irradiation

 

 

Slide 1 Slide 2 Slide 3 Slide 4

A 65-year-old woman has a 5-month history of increasing hip and thigh pain. She has severe pain with weight bearing on the right leg. Her plain radiographs (Slide 1) and T1- and T2-weighted coronal images (Slides 2 and 3) of her hip are presented. A biopsy is also performed (Slide 4). The cells directly responsible for the bone destruction are:

 

1) A. Tumor cells

3) C. Osteoclasts

2) B. Fibroblasts

5) E. Plasma cells

4) D. Histiocytes

 

The plain radiographs show an ill-defined permeative lesion in the proximal femur. The lateral cortex of the greater trochanter is thinned. The T1-weighted coronal images show a low-signal lesion in the entire greater trochanter extending to the base of the femoral neck. In the opposite proximal femur and acetabulum, there are multiple small low-signal lesions consistent with metastatic bone disease. The T2-weighted images show a very high-signal area in the greater trochanter. The inversion recovery image is sensitive to pathologic processes. Normal marrow will be low signal and pathologic lesions are high signal. The biopsy shows round epithelial cells forming glands. This pattern is characteristic of metastatic adenocarcinoma.

 

The bone destruction in metastatic bone disease is a direct result of osteoclastic resorption of the bone. The metastatic breast cells secrete a factor that activates and recruits osteoclasts that destroy the bone. Diphosphonate therapy (usually intravenous pamidronate) is given to virtually all women with metastatic breast carcinoma to stop the osteoclasts from resorbing the bone matrix.

 

 

 

 

 

 

Correct Answer: C. Osteoclasts 3620. (1085) Q9-1444:

 

Slide 1 Slide 2

A 45-year-old man has a 4-month history of severe pain in his shoulder. His plain radiograph (Slide 1) and biopsy (Slide 2) are presented. The most likely diagnosis is:

 

1) Malignant fibrous histiocytoma

3) Multiple myeloma

2) Pagetâs disease

5) Hyperparathyroidism (brown tumor)

4) Metastatic adenocarcinoma

 

The plain radiograph shows a destructive and purely lytic lesion. More than 50% of the cortical bone has been destroyed. This pattern of permeative bone destruction is consistent with a malignancy. The biopsy shows round, epithelial-like cells with clear cytoplasm. The cells are organized in clusters. This is metastatic clear cell carcinoma of the kidney.

 

 

This patient is prone to fracture. The humeral lesion meets the criteria of an impending fracture. Correct Answer: Metastatic adenocarcinoma

 

 

 

 

Slide 1 Slide 2

A 45-year-old man has a 4-month history of severe pain in his shoulder. His plain radiograph (Slide 1) and biopsy (Slide 2) are presented. The most appropriate treatment method is:

 

1) Shoulder disarticulation

3) External beam irradiation

2) Wide resection and allograft reconstruction

5) Neck exploration and parathyroidectomy

4) Internal fixation with external beam irradiation

 

The plain radiograph shows a destructive and purely lytic lesion. More than 50% of the cortical bone has been destroyed. This pattern of permeative bone destruction is consistent with a malignancy. The biopsy shows round, epithelial-like cells with clear cytoplasm. The cells are organized in clusters. This is metastatic clear cell carcinoma of the kidney.

 

This patient is prone to fracture. The humeral lesion meets the criteria of an impending fracture. The best treatment is internal fixation with an intramedullary rod and postoperative external beam irradiation. Plate fixation with methylmethacrylate supplementation would be another reasonable choice.

 

 

 

 

 

 

Correct Answer: Internal fixation with external beam irradiation 3622. (1087) Q9-1446:

 

Slide 1 Slide 2

A 70-year-old man has severe shoulder pain. A plain radiograph (Slide 1) and a biopsy specimen (Slide 2) are presented. The most likely diagnosis is:

 

1) Multiple myeloma

3) Bone island (enostosis)

2) Pagetâs disease

5) Osteosarcoma occurring in Pagetâs disease

4) Metastatic carcinoma

 

The plain radiograph of the humerus shows a densely sclerotic lesion. The cortices are intact and show no thickening or remodeling features that are characteristic of Pagetâs disease. The lesion has no apparent bone destruction. The biopsy specimen shows epithelial cells formed in clusters (or an organoid pattern). This is metastatic carcinoma.

 

Correct Answer: Metastatic carcinoma

 

 

 

 

 

Slide 1 Slide 2

A 70-year-old man has severe shoulder pain. A plain radiograph (Slide 1) and a biopsy specimen (Slide 2) are presented. The most appropriate treatment for the humeral lesion is:

 

1) Wide resection and allograft prosthetic reconstruction

3) Intramedullary rod fixation and external beam irradiation

2) Diphosphonate therapy

5) External beam irradiation

4) Subacromial cortisone injection

 

The plain radiograph of the humerus shows a densely sclerotic lesion. The cortices are intact and show no thickening or remodeling features that are characteristic of Pagetâs disease. The lesion has no apparent bone destruction. The biopsy specimen shows epithelial cells formed in clusters (or an organoid pattern). This is metastatic carcinoma.

 

The goals of treatment in metastatic disease are to relieve pain, prevent fracture, and maintain function. Purely blastic lesions have a low risk of fracture. This patient does not need prophylactic fixation. The best treatment choice is external beam irradiation alone.

 

 

 

 

 

 

Correct Answer: External beam irradiation 3624. (1132) Q9-1493:

 

Slide 1 Slide 2

A 66-year-old man has a 4-month history of hip discomfort. The plain radiographs are shown in Slide 1 and a biopsy is shown in Slide 2. The most likely diagnosis is:

 

1) Metastatic bone disease

3) Multiple myeloma

2) Lymphoma

5) Malignant fibrous histiocytoma

4) Chondrosarcoma

 

The plain radiograph of the pelvis shows a lytic lesion in the supra-acetabular area. A destructive lesion in the pelvis may be secondary to five common processes in patients who are 40 to 80 years of age, including metastatic bone disease, multiple myeloma, lymphoma, chondrosarcoma, and malignant fibrous histiocytoma. Often a biopsy is necessary to establish the diagnosis. The biopsy specimen shows plasma cells. The plasma cells have these characteristics:

 

 

Eccentrically placed nucleus

 

 

Peripheral clumping of the nuclear chromatin A perinuclear halo

 

The diagnosis is multiple myeloma. The treatment would be external beam irradiation. Correct Answer: Multiple myeloma

 

 

 

 

Slide 1 Slide 2 Slide 3

A 60-year-old man has a 6-month history of increasing knee pain. He has discomfort at night and at rest. His anteroposterior and lateral radiographs are shown in Slide 1, and low and high power biopsy specimens are shown in Slides 2 and 3. The most likely diagnosis is:

 

1) Metastatic bone disease

3) Malignant fibrous histiocytoma

2) Multiple myeloma

5) Dedifferentiated chondrosarcoma

4) Lymphoma

 

The plain radiographs show two major features. The proximal portion of the lesion (top half) shows an intramedullary lesion, which is mineralized. The mineralization shows rings and stipples. There is also thickening of the cortex of the femur. This thickening is circumferential on the medial, lateral, anterior, and posterior cortices. These features strongly suggest a cartilage tumor, principally a chondrosarcoma. The distal (lower half) of the lesion has a different pattern. The intramedullary lesion is completely lytic, and the lateral cortex has been destroyed. This bimorphic pattern is seen with dedifferentiated chondrosarcoma. The bimorphic pattern is a low-grade chondrosarcoma and a high-grade anaplastic spindle cell sarcoma.

 

The histologic appearance is also bimorphic. Notice the cartilage, which is relatively hypocellular, and the intimate admixture of a high-grade spindle cell sarcoma. The high-grade spindle sarcoma is characterized by pleomorphic and pyknotic nucleiâdark staining nuclear chromatin, and different sizes and shapes of the nuclei. The bimorphic pattern is low-grade cartilage admixed with high-grade spindle cell sarcoma. This high-grade spindle cell sarcoma can be osteosarcoma, malignant fibrous histiocytoma, or fibrosarcoma.

 

This tumor is a high-grade malignancy with a long-term survival of only 20% to 40%. The lungs are the most common site of metastases. Treatment is wide resection.

Correct Answer: Dedifferentiated chondrosarcoma

 

 

Slide 1 Slide 2 Slide 3

A 60-year-old man has a 6-month history of increasing knee pain. He has discomfort at night and at rest. His anteroposterior and lateral radiographs are shown in Slide 1, and low and high power biopsy specimens are shown in Slides 2 and 3. The treatment of this lesion would be:

 

1) Intramedullary rod fixation and external beam irradiation

3) Wide resection and custom prosthetic replacement

2) Curettage, cement augmentation and plate fixation

5) Neck exploration and parathyroidectomy

4) External beam irradiation alone

 

The plain radiographs show two major features. The proximal portion of the lesion (top half) shows an intramedullary lesion, which is mineralized. The mineralization shows rings and stipples. There is also thickening of the cortex of the femur. This thickening is circumferential on the medial, lateral, anterior, and posterior cortices. These features strongly suggest a cartilage tumor, principally a chondrosarcoma. The distal (lower half) of the lesion has a different pattern. The intramedullary lesion is completely lytic, and the lateral cortex has been destroyed. This bimorphic pattern is seen with dedifferentiated chondrosarcoma. The bimorphic pattern is a low-grade chondrosarcoma and a high-grade anaplastic spindle cell sarcoma.

 

The histologic appearance is also bimorphic. Notice the cartilage, which is relatively hypocellular, and the intimate admixture of a high-grade spindle cell sarcoma. The high-grade spindle sarcoma is characterized by pleomorphic and pyknotic nucleiâdark staining nuclear chromatin, and different sizes and shapes of the nuclei. The bimorphic pattern is low-grade cartilage admixed with high-grade spindle cell sarcoma. This high-grade spindle cell sarcoma can be osteosarcoma, malignant fibrous histiocytoma, or fibrosarcoma.

 

The treatment of dedifferentiated chondrosarcoma is wide resection and custom replacement. Amputation with a wide margin would be another option.

 

The prognosis is poor with a long-term survival of only 20% to 40%. The most common site of metastases is to the lungs. Correct Answer: Wide resection and custom prosthetic replacement

 

Slide 1 Slide 2 Slide 3

A 60-year-old man has a 6-month history of increasing knee pain. He has discomfort at night and at rest. His anteroposterior and lateral radiographs are shown in Slide 1, and low and high power biopsy specimens are shown in Slides 2 and 3. The prognosis for this patient would be:

 

1) Excellent (> 90% long-term survival)

3) Poor (< 25% long-term survival)

2) Good (> 50% long-term survival)

5) Dismal (no long-term survivors but prolonged disease course)

4) Dismal (no long-term survivors)

 

The plain radiographs show two major features. The proximal portion of the lesion (top half) shows an intramedullary lesion, which is mineralized. The mineralization shows rings and stipples. There is also thickening of the cortex of the femur. This thickening is circumferential on the medial, lateral, anterior, and posterior cortices. These features strongly suggest a cartilage tumor, principally a chondrosarcoma. The distal (lower half) of the lesion has a different pattern. The intramedullary lesion is completely lytic, and the lateral cortex has been destroyed. This bimorphic pattern is seen with dedifferentiated chondrosarcoma. The bimorphic pattern is a low-grade chondrosarcoma and a high-grade anaplastic spindle cell sarcoma.

 

The histologic appearance is also bimorphic. Notice the cartilage, which is relatively hypocellular, and the intimate admixture of a high-grade spindle cell sarcoma. The high-grade spindle sarcoma is characterized by pleomorphic and pyknotic nucleiâdark staining nuclear chromatin, and different sizes and shapes of the nuclei. The bimorphic pattern is low-grade cartilage admixed with high-grade spindle cell sarcoma. This high-grade spindle cell sarcoma can be osteosarcoma, malignant fibrous histiocytoma, or fibrosarcoma.

 

 

Dedifferentiated chondrosarcomas are very high-grade lesions with a poor prognosis. Long-term survival is < 25% of patients. Correct Answer: Poor (< 25% long-term survival)

 

Slide 1 Slide 2 Slide 3

A 60-year-old man has a 6-month history of increasing knee pain. He has discomfort at night and at rest. His anteroposterior and lateral radiographs are shown in Slide 1 and low and high power biopsy specimens in Slides 2 and 3. Chest radiograph and computerized tomography scans are normal. The stage of disease according to the system of the Musculoskeletal Tumor Society is:

 

1) Stage 1

3) Stage I

2) Stage 2

5) Stage III

4) Stage II

 

The plain radiographs show two major features. The proximal portion of the lesion (top half) shows an intramedullary lesion, which is mineralized. The mineralization shows rings and stipples. There is also thickening of the cortex of the femur. This thickening is circumferential on the medial, lateral, anterior, and posterior cortices. These features strongly suggest a cartilage tumor, principally a chondrosarcoma. The distal (lower half) of the lesion has a different pattern. The intramedullary lesion is completely lytic, and the lateral cortex has been destroyed. This bimorphic pattern is seen with dedifferentiated chondrosarcoma. The bimorphic pattern is a low-grade chondrosarcoma and a high-grade anaplastic spindle cell sarcoma.

 

The histologic appearance is also bimorphic. Notice the cartilage, which is relatively hypocellular, and the intimate admixture of a high-grade spindle cell sarcoma. The high-grade spindle sarcoma is characterized by pleomorphic and pyknotic nucleiâdark staining nuclear chromatin, and different sizes and shapes of the nuclei. The bimorphic pattern is low-grade cartilage admixed with high-grade spindle cell sarcoma. This high-grade spindle cell sarcoma can be osteosarcoma, malignant fibrous histiocytoma, or fibrosarcoma.

 

This patient has a high-grade sarcoma and the staging studies show no evidence of pulmonary metastases. The surgical stage would be stage II. If an magnetic resonance imaging scan showed soft tissue extension, then the stage would be Stage IIB.

Correct Answer: Stage II

 

 

Slide 1 Slide 2 Slide 3

A 60-year-old man has a 6-month history of increasing knee pain. He has discomfort at night and at rest. His anteroposterior and lateral radiographs are shown in Slide 1 and low and high power biopsy specimens in Slides 2 and 3. The most common site of metastases in this patient would be:

 

1) Other bones

3) Liver

2) Lymph nodes

5) Brain

4) Lung

 

The plain radiographs show two major features. The proximal portion of the lesion (top half) shows an intramedullary lesion, which is mineralized. The mineralization shows rings and stipples. There is also thickening of the cortex of the femur. This thickening is circumferential on the medial, lateral, anterior, and posterior cortices. These features strongly suggest a cartilage tumor, principally a chondrosarcoma. The distal (lower half) of the lesion has a different pattern. The intramedullary lesion is completely lytic, and the lateral cortex has been destroyed. This bimorphic pattern is seen with dedifferentiated chondrosarcoma. The bimorphic pattern is a low-grade chondrosarcoma and a high-grade anaplastic spindle cell sarcoma.

 

The histologic appearance is also bimorphic. Notice the cartilage, which is relatively hypocellular, and the intimate admixture of a high-grade spindle cell sarcoma. The high-grade spindle sarcoma is characterized by pleomorphic and pyknotic nucleiâdark staining nuclear chromatin, and different sizes and shapes of the nuclei. The bimorphic pattern is low-grade cartilage admixed with high-grade spindle cell sarcoma. This high-grade spindle cell sarcoma can be osteosarcoma, malignant fibrous histiocytoma, or fibrosarcoma.

 

Dedifferentiated chondrosarcomas are very lethal malignancies with only a 25% long-term survival rate. They most commonly metastasize to the lungs.

Correct Answer: Lung

 

 

 

Slide 1 Slide 2

A 66-year-old man has a 4-month history of hip discomfort. The plain radiographs are shown in Slide 1 and a biopsy is shown in Slide 2. The most appropriate treatment would be:

 

1) Wide resection of the acetabulum

3) Systemic chemotherapy

2) Curettage and bone grafting

5) Subacromial cortisone injection

4) Forequarter amputation

 

A destructive lesion in the pelvis may be secondary to five common processes in patients who are 40 to 80 years of age, including metastatic bone disease, multiple myeloma, lymphoma, chondrosarcoma, and malignant fibrous histiocytoma. Often a biopsy is necessary to establish the diagnosis. The biopsy specimen shows plasma cells. The plasma cells have these characteristics:

 

 

Eccentrically placed nucleus

 

 

Peripheral clumping of the nuclear chromatin A perinuclear halo

The diagnosis is multiple myeloma. The treatment of multiple myeloma is systemic chemotherapy (usually prednisone and an alkylating agent). External beam radiation is used for pain control and to stop lesion progression. When there is no risk of impending fracture, chemotherapy would be tried first.

 

Correct Answer: Systemic chemotherapy

 

 

 

Slide 1 Slide 2

A 66-year-old man has a 4-month history of hip discomfort. The plain radiographs are shown in Slide 1 and a biopsy is shown in Slide 2. Which of the following tests would be recommended to assess the extent of this condition:

 

1) Computerized tomography scan and technetium bone scan

3) Skeletal survey and bone marrow aspiration

2) Technetium bone scan alone

5) Positron emission tomography scan of the brain and computerized tomography of the chest

4) Computerized tomography of the chest and abdomen and technetium bone scan

 

A destructive lesion in the pelvis may be secondary to five common processes in patients who are 40 to 80 years of age, including metastatic bone disease, multiple myeloma, lymphoma, chondrosarcoma, and malignant fibrous histiocytoma. Often a biopsy is necessary to establish the diagnosis. The biopsy specimen shows plasma cells. The plasma cells have these characteristics:

 

 

Eccentrically placed nucleus

 

 

Peripheral clumping of the nuclear chromatin A perinuclear halo

The diagnosis is multiple myeloma. Staging is important and is performed with a skeletal survey, bone marrow biopsy, and chemistry studies including kidney function and serum calcium determination.

Technetium bone scans have a high false-negative rate and are not used in the staging process. Neither computerized tomography scans nor brain scans are routinely used.

Correct Answer: Skeletal survey and bone marrow aspiration

 

 

 

Slide 1 Slide 2

A 66-year-old man has a 4-month history of hip discomfort. The plain radiographs are shown in Slide 1 and a biopsy is shown in Slide 2. Which of the following tests would not be recommended to assess the extent of this condition (staging studies):

 

1) Bone marrow biopsy

3) Serum creatinine and blood urea nitrogen

2) Serum calcium level

5) Skeletal survey

4) Technetium bone scan

 

A destructive lesion in the pelvis may be secondary to five common processes in patients who are 40 to 80 years of age, including metastatic bone disease, multiple myeloma, lymphoma, chondrosarcoma, and malignant fibrous histiocytoma. Often a biopsy is necessary to establish the diagnosis. The biopsy specimen shows plasma cells. The plasma cells have these characteristics:

 

 

Eccentrically placed nucleus

 

 

Peripheral clumping of the nuclear chromatin A perinuclear halo

The diagnosis is multiple myeloma. Staging is important and is performed with a skeletal survey, bone marrow biopsy, and chemistry studies, including kidney function and serum calcium determination.

Technetium bone scans have a high false-negative rate and are not used in the staging process. Neither computerized tomography scans nor brain scans are routinely used.

Correct Answer: Technetium bone scan

 

 

 

Slide 1 Slide 2

A 66-year-old man has a 4-month history of hip discomfort. The plain radiographs are shown in Slide 1 and a biopsy is shown in Slide 2. Which of the following tests would be ordered to further confirm the diagnosis:

 

1) Serum protein electrophoresis

3) Urine hydroxy proline levels

2) Serum alkaline phosphatase level

5) Serum prostate-specific antigen level

4) Serum parathyroid hormone level

 

A destructive lesion in the pelvis may be secondary to five common processes in patients who are 40 to 80 years of age, including metastatic bone disease, multiple myeloma, lymphoma, chondrosarcoma, and malignant fibrous histiocytoma. Often a biopsy is necessary to establish the diagnosis. The biopsy specimen shows plasma cells. The plasma cells have these characteristics:

 

 

Eccentrically placed nucleus

 

 

Peripheral clumping of the nuclear chromatin A perinuclear halo

The diagnosis is multiple myeloma. Staging is important and is performed with a skeletal survey, bone marrow biopsy, hemoglobin level, and chemistry studies including kidney function and serum calcium determination. Serum protein electrophoresis is sensitive in 75% of patients, whereas urine immunoelectrophoresis is 95% sensitive.

 

Serum protein electrophoresis is performed to assess the amount of abnormal gammaglobulin in the serum. There are three major criteria in the diagnosis of multiple myeloma:

 

Plasmacytoma on tissue biopsy

 

Bone marrow plasmacytosis > 30% plasma cells

 

Monoclonal immunoglobulin spike on serum protein electrophoresis exceeding 3.5 g/dl for G peaks or 2.0 g/dl for A peaks or 1.0 g/24 hours of kappa or lamba light chain excretion on urine electrophoresis

Correct Answer: Serum protein electrophoresis

 

 

 

Slide 1 Slide 2

A 66-year-old man has a 4-month history of hip discomfort. The plain radiographs are shown in Slide 1 and a biopsy is shown in Slide 2. Which of the following tests will probably be abnormal:

 

1) Serum alkaline phosphatase levels

3) Serum parathyroid hormone level

2) Urine hydroxyproline levels

5) Serum prostate specific antigen level

4) Serum hemoglobin and erythrocyte sedimentation rate

 

A destructive lesion in the pelvis may be secondary to five common processes in patients who are 40 to 80 years of age, including metastatic bone disease, multiple myeloma, lymphoma, chondrosarcoma, and malignant fibrous histiocytoma. Open biopsy is often necessary to establish the diagnosis. The biopsy specimen shows plasma cells. The plasma cells have these characteristics:

 

 

Eccentrically placed nucleus

 

 

Peripheral clumping of the nuclear chromatin A perinuclear halo

The diagnosis is multiple myeloma. Staging is important and is performed with a skeletal survey, bone marrow biopsy, hemoglobin level, and chemistry studies including kidney function and serum calcium determination.

 

The serum hemoglobin and erythrocyte sedimentation rate will be abnormal in approximately two-thirds of patients. (Hemoglobin <12g/d/ l and ESR 750mm/h)Correct Answer: Serum hemoglobin and erythrocyte sedimentation rate

 

Slide 1 Slide 2 Slide 3 Slide 4

A 55-year-old man has a 4-month history of increasing knee pain. The plain radiographs are shown in Slide 1, and a coronal T1 weighted magnetic resonance scan is shown in Slide 2. A biopsy of the lesion is shown in Slides 3 and 4. The most likely diagnosis is:

 

1) Metastatic bone disease

3) Chondrosarcoma

2) Multiple myeloma

5) Malignant fibrous histiocytoma

4) Lymphoma

 

The plain radiograph shows a destructive lesion in the distal femoral metaphysis. Notice the lesion is purely lytic. The magnetic resonance imaging scan shows a very large lesion with destruction of the cortex and a large soft tissue mass. This radiograph appearance indicates a malignant lesion. The possibilities include a chondrosarcoma, malignant fibrous histiocytoma, metastasis, myeloma, and lymphoma. The biopsy specimen shows a storiform pattern with fascicles of spindle cells. The high power specimen shows spindle cells with pleomorphism, which are different nuclear sizes and shapes, and histiocytes, which are cells with round and vesicular nuclei. The diagnosis is malignant fibrous histiocytoma.

 

 

 

 

 

 

 

 

 

 

Correct Answer: Malignant fibrous histiocytoma 3636. (1144) Q9-1508:

 

Slide 1 Slide 2 Slide 3 Slide 4

A 55-year-old man has a 4-month history of increasing knee pain. The plain radiographs are shown in Slide 1 and a coronal T1 weighted magnetic resonance scan is shown in Slide 2. A biopsy of the lesion is shown in Slides 3 and 4. The most appropriate treatment method would be:

 

1) Curettage and cement augmentation

3) External beam irradiation alone

2) Curettage, cement augmentation, and external beam irradiation

5) Wide resection

4) External beam irradiation, diphosphonates, and chemotherapy

 

The plain radiograph shows a destructive lesion in the distal femoral metaphysis. Notice the lesion is purely lytic. The magnetic resonance imaging scan shows a very large lesion with destruction of the cortex and a large soft tissue mass. This radiographic appearance indicates a malignant lesion. The possibilities include a chondrosarcoma, malignant fibrous histiocytoma, metastasis, myeloma, and lymphoma. The biopsy specimen shows a storiform pattern with fascicles of spindle cells. The high power specimen shows spindle cells with pleomorphism, which are different nuclear sizes and shapes, and histiocytes, which are cells with round and vesicular nuclei. The diagnosis is malignant fibrous histiocytoma. The treatment of malignant fibrous histiocytoma is wide resection. Wide resection can be accomplished with limb salvage or amputation.

 

Correct Answer: Wide resection

 

 

Slide 1 Slide 2 Slide 3 Slide 4

A 55-year-old man has a 4-month history of increasing knee pain. The plain radiographs are shown in Slide 1 and a coronal T1 weighted magnetic resonance scan is shown in Slide 2. A biopsy of the lesion is shown in Slides 3 and 4. Which of the following studies should be done to aid in staging of the patientâs disease:

 

1) Skeletal survey

3) Bone marrow aspiration/biopsy

2) Serum protein electrophoresis

5) CT scan of the abdomen

4) Computed tomography (CT) scan of the chest

 

The plain radiograph shows a destructive lesion in the distal femoral metaphysis. Notice the lesion is purely lytic. The magnetic resonance imaging scan shows a very large lesion with destruction of the cortex and a large soft tissue mass. This radiograph appearance indicates a malignant lesion. The possibilities include a chondrosarcoma, malignant fibrous histiocytoma, metastasis, and lymphoma. The biopsy specimen shows a storiform pattern and fascicle of spindle cells. The high power specimen shows spindle cells with pleomorphism, which are different nuclear sizes and shapes, and histiocytes, which are cells with round and vesicular nuclei. The diagnosis is malignant fibrous histiocytoma. The most common site of metastases would be to the lungs. The best staging test would be a CT scan of the chest. Skeletal survey, serum protein electrophoresis, and bone marrow aspiration would be used for staging in multiple myeloma. Computed tomography of the chest and abdomen is used for staging in metastatic bone disease.

 

 

 

 

 

 

 

 

 

 

Correct Answer: Computed tomography (CT) scan of the chest 3638. (1146) Q9-1510:

 

Slide 1 Slide 2 Slide 3 Slide 4

A 65-year-old woman has a 5-month history of increasing hip and thigh pain. Her plain radiographs are shown in Slide 1. T1 and T2 weighted coronal images of her hip are shown in Slides 2 and 3 respectively. A biopsy is performed and is shown in Slide 4. The most likely diagnosis is:

 

1) Multiple myeloma

3) Metastatic adenocarcinoma

2) Lymphoma

5) Paget disease

4) Chondrosarcoma

 

The plain radiographs show an ill-defined permeative lesion in the proximal femur. The lateral cortex of the greater trochanter is thinned. The T1 weighted coronal images show a low signal lesion in the entire greater trochanter extending to the base of the femoral neck. Notice in the opposite proximal femur and acetabulum, there are multiple small low signal lesions consistent with metastatic bone disease. The T2 weighted images show a very high signal area in the greater trochanter. The inversion recovery image is very sensitive to pathologic processes. Normal marrow will be very low signal while pathologic lesions are high signal. The biopsy shows round epithelial cells forming glands. This pattern is that of metastatic adenocarcinoma.

 

Correct Answer: Metastatic adenocarcinoma

 

 

 

 

 

Slide 1 Slide 2 Slide 3 Slide 4

A 65-year-old woman has a 5-month history of increasing hip and thigh pain. Her plain radiographs are shown in Slide 1. She has severe pain with weight bearing on the right leg. T1 and T2 weighted coronal images of her hip are shown in Slides 2 and 3 respectively. A biopsy is performed and is shown in Slide 4. The most appropriate treatment would be:

 

1) Hip disarticulation

3) Wide resection and custom proximal femoral replacement

2) Neck exploration and removal of her parathyroid glands

5) Internal fixation followed by external beam irradiation

4) Crutch ambulation for 6 to 8 weeks

 

The plain radiographs show an ill-defined permeative lesion in the proximal femur. The lateral cortex of the greater trochanter is thinned. The T1 weighted coronal images show a low signal lesion in the entire greater trochanter extending to the base of the femoral neck. Notice in the opposite proximal femur and acetabulum, there are multiple small low signal lesions consistent with metastatic bone disease. The T2 weighted images show a very high signal area in the greater trochanter. The inversion recovery image is very sensitive to pathologic processes. Normal marrow will be very low signal while pathologic lesions are high signal. The biopsy shows round epithelial cells forming glands. This pattern is that of metastatic adenocarcinoma.

 

The treatment of metastatic breast carcinoma of bone is external beam irradiation to symptomatic lesions and diphosphonate therapy to retard the bone loss. Internal fixation is performed to prevent pathologic fracture. When patients have significant weight bearing pain, and a purely lytic lesion in the hip, the risk of fracture is high. The best treatment method for this patient is internal fixation followed by external beam irradiation.

 

Correct Answer: Internal fixation followed by external beam irradiation

 

 

 

 

 

Slide 1 Slide 2 Slide 3 Slide 4

A 65-year-old woman has a 5-month history of increasing hip and thigh pain. Her plain radiographs are shown in Slide 1. She has severe pain with weight bearing on the right leg. T1 and T2 weighted coronal images of her hip are shown in Slides 2 and 3 respectively. A biopsy is performed and is shown in Slide 4. The cells directly responsible for the bone destruction would be:

 

1) Tumor cells

3) Osteoclasts

2) Fibroblasts

5) Plasma cells

4) Histiocytes

 

The plain radiographs show an ill-defined permeative lesion in the proximal femur. The lateral cortex of the greater trochanter is thinned. The T1 weighted coronal images show a low signal lesion in the entire greater trochanter extending to the base of the femoral neck. Notice in the opposite proximal femur and acetabulum, there are multiple small low signal lesions consistent with metastatic bone disease. The T2 weighted images show a very high signal area in the greater trochanter. The inversion recovery image is very sensitive to pathologic processes. Normal marrow will be very low signal while pathologic lesions are high signal. The biopsy shows round epithelial cells forming glands. This pattern is that of metastatic adenocarcinoma.

 

The bone destruction in metastatic bone disease is a direct result of osteoclastic resorption of the bone. The metastatic breast cells secrete a factor that activates and recruits osteoclasts, which then destroy the bone. Diphosphonate therapy (usually intravenous pamidronate) is given to virtually all women with metastatic breast carcinoma to bone to stop the osteoclasts from resorbing the bone matrix.

 

Correct Answer: Osteoclasts

 

 

Slide 1 Slide 2 Slide 3 Slide 4

 

 

 

Slide 5 Slide 6

A 50-year-old woman has had severe hip pain for 4 months. Her plain radiographs are shown in Slide 1. The technetium bone scan, computerized tomography scan, and coronal T1 and T2 weighted magnetic resonance imaging scans are shown in Slides 2, 3, and 4 respectively. A needle biopsy is performed and shown in Slides 5 and 6. The most likely diagnosis is:

 

1) Metastatic adenocarcinoma

3) Lymphoma

2) Multiple myeloma

5) Malignant fibrous histiocytoma

4) Chondrosarcoma

 

The plain radiographs show a purely lytic destructive lesion, which is poorly marginated. The technetium bone scan does not show any major uptake. The computerized tomography scan shows purely lytic bone destruction with breakthrough of the cortical bone. Complete destruction of the cortical bone is very suggestive of a malignancy. The magnetic resonance imaging scan shows a lesion that is homogenously low on T1 weighted images and high on T2 weighted images. One cannot make a definitive diagnosis based upon the radiographic features. The most common malignancies in this age group are:

 

 

 

 

 

Metastatic bone disease Multiple myeloma Lymphoma Chondrosarcoma

 

Malignant fibrous histiocytoma

The biopsy specimen shows round epithelial appearing cells that are grouped in clusters. The cells are arranged in a fibrous background. The diagnosis is metastatic bone disease.

Correct Answer: Metastatic adenocarcinoma

 

 

Slide 1 Slide 2 Slide 3 Slide 4

 

 

 

Slide 5 Slide 6

A 50-year-old woman has had severe hip pain for 4 months. Her plain radiographs are shown in Slide 1. The technetium bone scan, computerized tomography scan, and coronal T1 and T2 weighted magnetic resonance imaging scans are shown in Slides 2, 3, and 4 respectively. A needle biopsy is performed and shown in Slides 5 and 6. Which of the following would be the most appropriate treatment regimen:

 

1) Wide resection and allograft acetabular reconstruction

3) Preoperative chemotherapy followed by wide resection

2) Internal hemi-pelvectomy alone

5) Hemi-pelvectomy

4) Acetabular reconstruction followed by external beam irradiation

 

The plain radiographs show a purely lytic destructive lesion, which is poorly marginated. The technetium bone scan does not show any major uptake. The computerized tomography scan shows purely lytic bone destruction with breakthrough of the cortical bone. Complete destruction of the cortical bone is very suggestive of a malignancy. The magnetic resonance imaging scan shows a lesion that is homogenously low on T1 weighted images and high on T2 weighted images. One cannot make a definitive diagnosis based upon the radiographic features. The most common malignancies in this age group are:

 

 

 

 

 

Metastatic bone disease Multiple myeloma Lymphoma Chondrosarcoma

 

Malignant fibrous histiocytoma

The biopsy specimen shows round epithelial-appearing cells that are grouped in clusters. The cells are arranged in a fibrous background. The diagnosis is metastatic bone disease.

The treatment of metatastic bone disease is either internal fixation with postoperative external beam irradiation or external beam irradiation alone. In this case, there is very extensive destruction and fracture is imminent. The treatment of choice would be acetabular reconstruction followed by external beam irradiation.

 

Correct Answer: Acetabular reconstruction followed by external beam irradiation

 

 

Slide 1 Slide 2 Slide 3 Slide 4

A 50-year-old woman has had severe hip pain for 4 months. Her plain radiographs are shown in Slide 1. The technetium bone scan, computerized tomography scan, and coronal T1 and T2 weighted magnetic resonance imaging scans are shown in Slides 2, 3, and 4 respectively. The most likely diagnosis based upon the radiographs would be:

 

1) Septic arthritis

3) Stress fracture

2) Osteomyelitis of the acetabulum

5) Malignant neoplasm

4) Lytic phase of Paget disease

 

The plain radiographs show a purely lytic destructive lesion, which is poorly marginated. The technetium bone scan does not show any major uptake. The computerized tomography scan shows purely lytic bone destruction with breakthrough of the cortical bone. Complete destruction of the cortical bone is very suggestive of a malignancy. The magnetic resonance imaging scan shows a lesion that is homogenously low on T1 weighted images and high on T2 weighted images. One cannot make a definitive diagnosis based upon the radiographic features; however, it is highly likely that this lesion is malignant. The cortical bone destruction is highly suggestive of a malignancy. The most common malignancies in this age group are:

 

 

 

 

 

Metastatic bone disease Multiple myeloma Lymphoma Chondrosarcoma

 

 

Malignant fibrous histiocytoma Correct Answer: Malignant neoplasm

 

Slide 1 Slide 2 Slide 3 Slide 4

 

 

 

Slide 5 Slide 6

A 50-year-old woman has had severe hip pain for 4 months. Her plain radiographs are shown in Slide 1. The technetium bone scan, computerized tomography scan, and coronal T1 and T2 weighted magnetic resonance imaging scans are shown in Slides 2, 3, and 4 respectively. A needle biopsy is performed and shown in Slides 5 and 6. The cells directly responsible for the bone destruction would be:

 

1) Tumor cells

3) Fibroblasts

2) Plasma cells

5) Mast cells

4) Osteoclasts

 

The plain radiographs show a purely lytic destructive lesion, which is poorly marginated. The technetium bone scan does not show any major uptake. The computerized tomography scan shows purely lytic bone destruction with breakthrough of the cortical bone. Complete destruction of the cortical bone is very suggestive of a malignancy. The magnetic resonance imaging scan shows a lesion that is homogenously low on T1 weighted images and high on T2 weighted images. One cannot make a definitive diagnosis based upon the radiographic features, however it is highly likely that this lesion is malignant. The biopsy specimen shows round epithelial-appearing cells that are grouped in clusters. The cells are arranged in a fibrous background. The diagnosis is metastatic bone disease.

 

The bone destruction in metastatic bone disease is a direct result of osteoclastic resorption of the bone. The metastatic breast cells secrete a factor that activates and recruits osteoclasts which then destroy the bone. Diphosphonate therapy (usually intravenous pamidronate) is given to virtually all women with metastatic breast carcinoma to bone to stop the osteoclasts from resorbing the bone matrix.

 

Correct Answer: Osteoclasts

 

 

Slide 1 Slide 2 Slide 3

A 60-year-old woman has severe knee pain that has been increasing for 6 months. The anteroposterior radiograph and T2 weighted coronal magnetic resonance imaging scan are shown in Slides 1 and 2. The biopsy specimen is shown in Slide 3. The most likely diagnosis is:

 

1) Metastatic bone disease

3) Lymphoma

2) Multiple myeloma

5) Malignant fibrous histiocytoma

4) Chondrosarcoma

 

The anteroposterior radiograph shows both arthritic changes in the medial compartment and a lytic destructive lesion in the lateral tibial plateau. Note the lucent area laterally and the area of increased density medially. The T2 weighted coronal magnetic resonance imaging scan shows complete marrow replacement of the proximal tibial plateau. The normal marrow is dark (low signal) and the tumor is high signal (white). The biopsy shows a mixture of large and small blue cells. This is the histologic appearance of lymphoma. Note that the other choices have characteristic histologic appearances.

 

 

Metastatic bone disease - epithelial cells organized in clusters in a fibrous background

 

Multiple myeloma - sheets of plasma cells, that have an eccentric nucleus with peripheral chromatin. There is also peri nuclear clearing or a halo where the Golgi apparatus is located

 

Chondrosarcoma - atypical chondrocytes in a blue chondroid matrix

 

 

Malignant fibrous histiocytoma - fascicles of fibroblasts and histiocytes in a storiform pattern Correct Answer: Lymphoma

 

Slide 1 Slide 2 Slide 3

A 60-year-old woman has severe knee pain that has been increasing for 6 months. The anteroposterior radiograph and T2 weighted coronal magnetic resonance imaging scan are shown in Slides 1 and 2. The biopsy specimen is shown in Slide 3. The most appropriate treatment method would be:

 

1) Wide resection and custom arthroplasty

3) Curettage and cement augmentation

2) Above knee amputation

5) Curettage and bone grafting

4) External beam irradiation and chemotherapy

 

The anteroposterior radiograph shows both arthritic changes in the medial compartment and a lytic destructive lesion in the lateral tibial plateau. Note the lucent area laterally and the area of increased density medially. The increased bone density gives a mottled appearance to the bone. The T2 weighted coronal magnetic resonance imaging scan shows complete marrow replacement of the proximal tibial plateau. The normal marrow is dark (low signal) and the tumor is high signal (white). The biopsy shows a mixture of large and small blue cells. This is the histologic appearance of lymphoma. One should remember the characteristic histologic appearances of bone tumors in adults.

 

 

Metastatic bone disease - epithelial cells organized in clusters in a fibrous background

 

Multiple myeloma - sheets of plasma cells, that have an eccentric nucleus with peripheral chromatin. There is also peri nuclear clearing or a halo where the Golgi apparatus is located

 

Chondrosarcoma - atypical chondrocytes in a blue chondroid matrix

 

Malignant fibrous histiocytoma - fascicles of fibroblasts and histiocytes in a storiform pattern

Most lymphomas of bone are large cell lymphomas and are treated with both external beam irradiation and chemotherapy. Surgical resection is usually not performed.

Correct Answer: External beam irradiation and chemotherapy

 

 

 

 

 

Slide 1 Slide 2 Slide 3

A 45-year-old woman sustained minor trauma to her hand while closing a window. Her plain radiograph and technetium bone scan (Slides 1 and 2) are presented. A biopsy is also performed (Slide 3). The most likely diagnosis is:

 

1) Grade 1 chondrosarcoma

3) Giant cell tumor

2) Enchondroma

5) Metastatic breast carcinoma

4) Giant cell reparative granuloma

 

The radiographs show a lucent lesion. The cortices are also thinned. These findings are diagnostic of an enchondroma of the small tubular bones of the hands. In a long bone, this degree of cortical change is diagnostic of a low-grade chondrosarcoma. In the small tubular bones of the hands and feet, these findings are consistent with an enchondroma. The bone scan shows increased uptake. The increased uptake has only limited significance in intramedullary cartilage lesions. Enchondromas may show little, moderate, or heavy uptake. The biopsy specimen shows chondroblasts with blue matrix of cartilage.

 

When asymptomatic, these lesions are treated with observation, while symptomatic lesions are treated with curettage and grafting.

 

 

 

 

 

 

 

Correct Answer: Enchondroma 3648. (1156) Q9-1520:

 

Slide 1 Slide 2 Slide 3

A 45-year-old woman sustained minor trauma to her hand while closing a window. Her plain radiograph and technetium bone scan (Slides 1 and 2) are presented. A biopsy is also performed (Slide 3). The most appropriate treatment would be:

 

1) External beam irradiation

3) Amputation through the mid aspect of the proximal phalanx

2) External beam irradiation and systemic chemotherapy

5) Curettage and bone grafting

4) Thumb ray amputation

 

The radiographs show a lucent lesion. The cortices are also thinned. These findings are diagnostic of an enchondroma of the small tubular bones of the hands. The treatment of enchondromas is curettage and bone grafting. If the enchondroma is asymptomatic, observation would also be an acceptable treatment method.

 

Correct Answer: Curettage and bone grafting

 

 

Slide 1 Slide 2 Slide 3

A 28-year-old woman has a 4-month history of increasing shoulder pain. Her plain radiograph is shown in Slide 1. A biopsy specimen is shown in Slides 2 and 3. The most likely diagnosis is:

 

1) Aneurysmal bone cyst

3) Telangiectatic osteosarcoma

2) Chondroblastoma

5) Unicameral bone cyst

4) Giant cell tumor

 

The plain radiograph shows a destructive lesion. The lesion is purely lytic, expansile, and there is a pathologic fracture. The lesion has destroyed the metaphysis and extended into the epiphysis.

 

From a radiographic perspective, this lesion could be: A telangiectatic osteosarcoma

 

 

An aneurysmal bone cyst A giant cell tumor

Unicameral bone cysts are rare in skeletally mature patients and the cortices are thinned but intact. Chondroblastomas are usually not this destructive.

The biopsy specimen shows a uniform distribution of multinucleated giant cells in a field of mononuclear cells. The diagnosis is giant cell tumor of bone.

Correct Answer: Giant cell tumor

 

 

Slide 1 Slide 2 Slide 3

A 28-year-old woman has a 4-month history of increasing shoulder pain. Her plain radiograph is shown in Slide 1. A biopsy specimen is shown in Slides 2 and 3. Which of the following is the best treatment option:

 

1) Preoperative chemotherapy followed by resection/reconstruction

3) Resection/reconstruction followed by chemotherapy

2) Resection/reconstruction followed by external beam irradiation

5) Amputation

4) Resection/reconstruction

 

The plain radiograph shows a destructive lesion. The lesion is purely lytic, expansile, and there is a pathologic fracture. The lesion has destroyed the metaphysis and extended into the epiphysis.

 

From a radiographic perspective, this lesion could be: A telangiectatic osteosarcoma

 

 

An aneurysmal bone cyst A giant cell tumor

The biopsy specimen shows a uniform distribution of multinucleated giant cells in a field of mononuclear cells. The diagnosis is giant cell tumor of bone.

Treatment of this lesion is resection followed by reconstruction. The reconstructive options include an osteoarticular allograft, allograft prosthetic composite, or a custom prosthesis. There is too much bone destruction to perform a curettage procedure.

Correct Answer: Resection/reconstruction

 

 

Slide 1 Slide 2 Slide 3

A 28-year-old woman has a 4-month history of increasing shoulder pain. Her plain radiograph is shown in Slide 1. A biopsy specimen is shown in Slides 2 and 3. Which of the following is the correct stage according to the system of the Musculoskeletal Tumor Society:

 

1) Stage 1

3) Stage 3

2) Stage 2

5) Stage II

4) Stage I

 

The plain radiograph shows a destructive lesion. The lesion is purely lytic, expansile, and there is a pathologic fracture. The lesion has destroyed the metaphysis and extended into the epiphysis.

 

From a radiographic perspective, this lesion could be: A telangiectatic osteosarcoma

 

 

An aneurysmal bone cyst A giant cell tumor

The biopsy specimen shows a uniform distribution of multinucleated giant cells in a field of mononuclear cells. The diagnosis is giant cell tumor of bone.

This lesion is an aggressive benign tumor â Stage 3. Remember the Musculoskeletal Tumor Society staging system:

Stage 1 Benign inactive

Stage 2 Benign active

Stage 3 Benign aggressive Â

Stage I Low grade malignant Stage II High grade malignant Stage III Metastases to any site Correct Answer: Stage 3

 

 

Slide 1 Slide 2 Slide 3

A 28-year-old woman has a 4-month history of increasing shoulder pain. Her plain radiograph is shown in Slide 1. A biopsy specimen is shown in Slides 2 and 3. Which of the following is the correct risk of pulmonary metastases for this lesion:

 

1) 0%

3) 10% to 30%

2) 2% to 5%

5) 75%

4) 30% to 50%

 

The plain radiograph shows a destructive lesion. The lesion is purely lytic, expansile, and there is a pathologic fracture. The lesion has destroyed the metaphysis and extended into the epiphysis.

 

From a radiographic perspective, this lesion could be: A telangiectatic osteosarcoma

 

 

An aneurysmal bone cyst A giant cell tumor

The biopsy specimen shows a uniform distribution of multinucleated giant cells in a field of mononuclear cells. The diagnosis is giant cell tumor of bone.

The risk of pulmonary metastases in giant cell tumor of bone is 2% to 5%. This phenomenon is called benign metastasizing giant cell tumor. The mortality rate from these metastases is 10% to 20%. Spontaneous regression of the pulmonary lesions may occur.

 

Correct Answer: 2% to 5%

 

 

Slide 5

A 15-year-old girl has a 4-month history of increasing wrist pain. Her clinical photograph and plain radiograph are shown in Slides 1 and 2. An axial T1-weighted image and sagittal T2-weighted image are shown in Slides 3 and 4. A biopsy specimen is shown in Slide 5. The most likely diagnosis is:

 

1) Unicameral bone cyst

3) Chondroblastoma

2) Aneurysmal bone cyst

5) Giant cell tumor

4) Telangiectatic osteosarcoma

 

The plain radiograph shows a destructive lesion. The lesion is purely lytic, expansile, and there is a pathologic fracture. The lesion has destroyed the metaphysis and extended into the epiphysis.

 

From a radiographic perspective, this lesion could be: A telangiectatic osteosarcoma

 

 

An aneurysmal bone cyst A giant cell tumor

Unicameral bone cysts are rare in skeletally mature patients and the cortices are thinned but intact. Chondroblastomas are usually not this destructive.

The biopsy specimen shows a uniform distribution of multinucleated giant cells in a field of mononuclear cells. The diagnosis is giant cell tumor of bone.

Correct Answer: Giant cell tumor

 

 

Slide 5

A 15-year-old girl has a 4-month history of increasing wrist pain. Her clinical photograph and plain radiograph are shown in Slides 1 and 2. An axial T1-weighted image and sagittal T2-weighted image are shown in Slides 3 and 4. A biopsy specimen is shown in Slide 5. Which of the following is the best treatment option:

 

1) Preoperative chemotherapy followed by resection/reconstruction

3) Resection/reconstruction followed by chemotherapy

2) Resection/reconstruction followed by external beam irradiation

5) Amputation

4) Resection/reconstruction

 

The plain radiograph shows a destructive lesion. The lesion is purely lytic, expansile, and there is a pathologic fracture. The lesion has destroyed the metaphysis and extended into the epiphysis.

 

From a radiographic perspective, this lesion could be: A telangiectatic osteosarcoma

 

 

An aneurysmal bone cyst A giant cell tumor

The biopsy specimen shows a uniform distribution of multinucleated giant cells in a field of mononuclear cells. The diagnosis is giant cell tumor of bone.

Treatment of this lesion is resection followed by reconstruction. The reconstructive options include an osteoarticular allograft, allograft fusion, or a fibular reconstruction. There is too much bone destruction to perform a curettage procedure.

Correct Answer: Resection/reconstruction

 

 

Slide 5

A 15-year-old girl has a 4-month history of increasing wrist pain. Her clinical photograph and plain radiograph are shown in Slides 1 and 2. An axial T1-weighted image and sagittal T2-weighted image are shown in Slides 3 and 4. A biopsy specimen is shown in Slide 5. Which of the following is the correct stage according to the system of the Musculoskeletal Tumor Society:

 

1) Stage 1

3) Stage 3

2) Stage 2

5) Stage II

4) Stage I

 

The plain radiograph shows a destructive lesion. The lesion is purely lytic, expansile, and there is a pathologic fracture. The lesion has destroyed the metaphysis and extended into the epiphysis.

 

From a radiographic perspective, this lesion could be: A telangiectatic osteosarcoma

 

 

An aneurysmal bone cyst A giant cell tumor

The biopsy specimen shows a uniform distribution of multinucleated giant cells in a field of mononuclear cells. The diagnosis is giant cell tumor of bone.

This lesion is an aggressive benign tumor â Stage 3. Remember the Musculoskeletal Tumor Society staging system:

Stage 1 Benign inactive

Stage 2 Benign active

Stage 3 Benign aggressive Â

Stage I Low grade malignant Stage II High grade malignant Stage III Metastases to any site Correct Answer: Stage 3

 

 

Slide 5

A 15-year-old girl has a 4-month history of increasing wrist pain. Her clinical photograph and plain radiograph are shown in Slides 1 and 2. An axial T1-weighted image and sagittal T2-weighted image are shown in Slides 3 and 4. A biopsy specimen is shown in Slide 5. Which of the following is the correct risk of pulmonary metastases for this lesion:

 

1) 0%

3) 10% to 30%

2) 2% to 5%

5) 75%

4) 30% to 50%

 

The plain radiograph shows a destructive lesion. The lesion is purely lytic, expansile, and there is a pathologic fracture. The lesion has destroyed the metaphysis and extended into the epiphysis.

 

From a radiographic perspective, this lesion could be: A telangiectatic osteosarcoma

 

 

An aneurysmal bone cyst A giant cell tumor

The biopsy specimen shows a uniform distribution of multinucleated giant cells in a field of mononuclear cells. The diagnosis is giant cell tumor of bone.

The risk of pulmonary metastases in giant cell tumor of bone is 2% to 5%. This phenomenon is called benign metastazing giant cell tumor. The mortality rate from these metastases is 10% to 20%. Spontaneous regression of the pulmonary lesions may occur.

 

Correct Answer: 2% to 5%

 

 

 

 

 

Slide 1

A 28-year-old woman has a destructive distal radius lesion. A biopsy specimen is shown in Slide 1. The most likely diagnosis is:

 

1) Aneurysmal bone cyst

3) Telangiectatic osteosarcoma

2) Chondroblastoma

5) Unicameral bone cyst

4) Giant cell tumor

 

The biopsy specimen shows a uniform distribution of multinucleated giant cells in a field of mononuclear cells. There are no atypical nuclear features in the mononuclear cells or the giant cells. The diagnosis is giant cell tumor of bone.Correct Answer: Giant cell tumor

 

 

3658. (1283) Q9-1653:

The cell of origin for malignant fibrous histiocytoma of bone is most likely:

 

1) A monocyte-macrophage

3) A chondroblast

2) A myofibroblast or fibroblast

5) Hematopoietic

4) An osteoblast

 

Malignant fibrous histiocytoma is an undifferentiated sarcoma. Tissue cultures show phenotypic features most consistent with myofibroblastic differentiation.Correct Answer: A myofibroblast or fibroblast

 

 

3659. (1284) Q9-1654:

Which of the following describes the histologic features of malignant fibrous histiocytoma of bone:

 

1) Fascicles of pleomorphic fibroblasts in a herringbone pattern

3) Pleomorphic spindle cells with direct production of osteoid

2) Spindle cells and pleomorphic cells in a storiform pattern

5) Low grade cartilage admixed with undifferentiated spindle cell sarcoma

4) Chondrocytes with round, darkly staining nuclei in a chondroid matrix

 

Malignant fibrous histiocytoma of bone is histologically characterized by spindle cells and pleomorphic cells in a storiform pattern. The pleomorphic cells display focal myofibroblastic differentiation.Correct Answer: Spindle cells and pleomorphic cells in a storiform pattern

 

 

3660. (1285) Q9-1655:

Which of the following is the most common location of malignant fibrous histiocytoma:

 

1) Pelvis

3) Femur

2) Tibia

5) Skull

4) Humerus

 

Malignant fibrous histiocytoma has a similar distribution to other malignant bone tumors. The knee is the most common region affected, and the femur is the most common bone.Correct Answer: Femur

 

Which of the following describes the most common radiographic features of malignant fibrous histiocytoma:

 

1) Metaphyseal, lytic with sclerotic rim, and thinned but intact overlying cortex

3) Lytic bone destruction with cortical disruption

2) Prominent erosions with matrix mineralization (stipples and rings)

5) Discrete lytic punched out lesions in multiple bones

4) Mixed pattern of bone destruction and formation with cortical thickening

 

The radiographic features of malignant fibrous histiocytoma of bone are nonspecific pattern of bone destruction. Common features include:

 

 

 

Lytic bone destruction Cortical destruction Soft tissue mass

 

The other answers describe specific lesions:

 

 

Nonossifying fibroma â Metaphyseal, lytic with sclerotic rim, and thinned but intact overlying cortex Chondrosarcoma â Prominent erosions with matrix mineralization (stipples and rings)

 

 

Lymphoma â Mixed pattern of bone destruction and formation with cortical thickening Myeloma â Discrete lytic punched out lesions in multiple bones

Correct Answer: Lytic bone destruction with cortical disruption

 

 

3662. (1287) Q9-1657:

Which of the following are the most common sites of metastases in patients with malignant fibrous histiocytoma of bone:

 

1) Bones

3) Lungs

2) Lymph nodes

5) Bones and lymph nodes

4) Lungs and lymph nodes

 

The most common site of metastases in malignant fibrous histiocytoma is to the lungs. Metastases to lymph nodes are rare.Correct Answer: Lungs

 

 

3663. (1288) Q9-1658:

Which of the following malignancies often occurs in a preexisting benign bone lesion:

 

1) Osteosarcoma

3) Fibrosarcoma

2) Angiosarcoma

5) Ewing tumor

4) Malignant fibrous histiocytoma

 

Malignant fibrous histiocytoma (MFH) is the most common sarcoma to occur in a preexisting benign bone lesion. Approximately 20% of MFHs occur in a preexisting benign bone condition such as bone infarcts, irradiated bone, fibrous dysplasia, and Pagetâs disease.Correct Answer: Malignant fibrous histiocytoma

 

 

Slide 1 Slide 2 Slide 3

A 45-year-old woman has hip discomfort with prolonged sitting, and she feels a mass. The plain radiographs and computerized tomography scan are shown in Slides 1, 2, and 3. The most likely diagnosis is:

 

1) Parosteal osteosarcoma

3) Osteochondroma

2) Periosteal chondroma

5) Secondary chondrosarcoma

4) Heterotopic ossification

 

The anteroposterior radiograph shows a lesion in the greater trochanter, and the oblique radiograph shows a bony mass projecting from the proximal femur. The computerized tomography scan shows a lesion arising from the posterior cortical surface, which shares its cortices with the host bone. In addition, the medullary cavity of the proximal femur flows into the lesion. These findings are consistent with an osteochondroma. There is no evidence of malignant change.

 

The radiographic findings of an osteochondroma are:

 

 

Underlying bone and the osteochondroma have continuous cortices. Medullary cavity of the bone and osteochondroma are continuous.

 

Secondary chondrosarcomas may develop in an osteochondroma. The radiographic features are: Destruction of the underlying subchondral cortex of the osteochondroma

 

Lytic destruction of the cartilage cap

 

 

Formation of a soft tissue mass (often calcified) Correct Answer: Osteochondroma

 

Slide 1 Slide 2 Slide 3

A 45-year-old woman has hip discomfort with prolonged sitting, and she feels a mass. The plain radiographs and computerized tomography scan are shown in Slides 1, 2, and 3. Her symptoms have increased, and she desires surgical treatment. The most appropriate treatment is:

 

1) Needle biopsy

3) Removal of the mass (excisional biopsy)

2) Incisional biopsy

5) Resection/proximal femoral replacement

4) Intramedullary rod fixation followed by irradiation

 

The anteroposterior radiograph shows a lesion in the greater trochanter, and the oblique radiograph shows a bony mass projecting from the proximal femur. The computerized tomography scan shows a lesion arising from the posterior cortical surface, which shares its cortices with the host bone. In addition, the medullary cavity of the proximal femur flows into the lesion. These findings are consistent with an osteochondroma. There is no evidence of malignant change.

 

The radiographic findings of an osteochondroma are:

 

 

Underlying bone and the osteochondroma have continuous cortices. Medullary cavity of the bone and osteochondroma are continuous.

 

Secondary chondrosarcomas may develop in an osteochondroma. The radiographic features are: Destruction of the underlying subchondral cortex of the osteochondroma

 

Lytic destruction of the cartilage cap

 

Formation of a soft tissue mass (often calcified)

 

The most appropriate treatment is removal of the mass through a posterior approach. Internal fixation may not be necessary. Correct Answer: Removal of the mass (excisional biopsy)

 

 

 

Slide 1 Slide 2 Slide 3 Slide 4

 

 

 

Slide 5

A 45-year-old woman has hip discomfort with prolonged sitting and feels a mass. The plain radiographs, and computerized tomography scan are shown in Slides 1, 2, and 3. Her symptoms have increased and she desires surgical treatment. She undergoes surgical removal and her biopsy specimens are shown in Slides 4 and 5. The diagnosis is:

 

1) Mesenchymal chondrosarcoma

3) Low grade chondrosarcoma

2) Dedifferentiated chondrosarcoma

5) Osteochondroma

4) Parosteal osteosarcoma

 

The anteroposterior radiograph shows a lesion in the greater trochanter, and the oblique radiograph shows a bony mass projecting from the proximal femur. The computerized tomography scan shows a lesion arising from the posterior cortical surface, which shares its cortices with the host bone. In addition, the medullary cavity of the proximal femur flows into the lesion. These findings are consistent with an osteochondroma. There is no evidence of malignant change.

 

The radiographic findings of an osteochondroma are:

 

 

Underlying bone and the osteochondroma have continuous cortices. Medullary cavity of the bone and osteochondroma are continuous.

 

Secondary chondrosarcomas may develop in an osteochondroma. The radiographic features are: Destruction of the underlying subchondral cortex of the osteochondroma

 

Lytic destruction of the cartilage cap

 

Formation of a soft tissue mass (often calcified)

The biopsy specimen shows a lesion with a thin cartilage cap. The high power view shows the typical features of an osteochondroma with a linear arrangement of the chondrocytes.

 

The lesion is benign. Observation is all that is required. Correct Answer: Osteochondroma

 

 

 

 

Slide 1 Slide 2

An 18-year-old woman has a hard bony mass over her distal right femur. The plain radiographs showed that this lesion was arising from the surface of the bone. The mass is removed, and the biopsy specimens are shown in Slides 1 and 2. The most likely diagnosis is:

 

1) Parosteal osteosarcoma

3) Heterotopic ossification

2) Periosteal osteosarcoma

5) Osteochondroma

4) Perisoteal chondroma

 

The biopsy specimen shows a lesion with a thin cartilage cap. The high power view shows the typical features of an osteochondroma with a linear arrangement of the chondrocytes. This lesion is an osteochondroma.Correct Answer: Osteochondroma

 

 

3668. (1293) Q9-1663:

 

 

 

Slide 1 Slide 2

An 18-year-old woman has a hard bony mass over her distal right femur. The plain radiographs showed that this lesion was arising from the surface of the bone. The mass is removed, and the biopsy specimens are shown in Slides 1 and 2. Which of the following is the correct treatment:

 

1) Observation

3) Radiation therapy

2) Chemotherapy

5) Chemotherapy and radiation

4) Repeat excision and radiation

 

The biopsy specimen shows a lesion with a thin cartilage cap. The high power view shows the typical features of an osteochondroma with a linear arrangement of the chondrocytes. This lesion is an osteochondroma.

 

Observation is the only treatment needed. Correct Answer: Observation

 

 

 

 

Slide 1 Slide 2

An 18-year-old woman has a hard mass over her distal femur. The mass is removed, and the biopsy specimens are shown in Slides 1 and 2. The most likely diagnosis is:

 

1) Parosteal osteosarcoma

3) Periosteal chondroma

2) Periosteal osteosarcoma

5) Heterotopic ossification

4) Osteochondroma

 

The biopsy specimen shows a lesion with a thin cartilage cap. Notice the blue cartilage matrix in the cartilage cap. The high power view shows the typical features of an osteochondroma with a linear arrangement of the chondrocytes. This lesion is an osteochondroma.Correct Answer: Osteochondroma

 

 

3670. (1295) Q9-1665:

 

 

 

Slide 1 Slide 2

An 18-year-old woman has a hard mass over her distal femur. The mass is removed, and the biopsy specimens are shown in Slides 1 and 2. The most appropriate treatment is:

 

1) Observation

3) Chemotherapy

2) External beam irradiation

5) Amputation

4) External beam irradiation and chemotherapy

 

The biopsy specimen shows a lesion with a thin cartilage cap. Notice the blue cartilage matrix in the cartilage cap. The high power view shows the typical features of an osteochondroma with a linear arrangement of the chondrocytes.

 

Observation is the most appropriate treatment. Correct Answer: Observation

 

Slide 1 Slide 2 Slide 3 Slide 4

A 15-year-old boy has a hard mass over his distal femur. He has pain when playing sporting activities. His plain radiographs are shown in Slides 1 and 2. The axial T1- and T2-weighted images are shown in Slides 3 and 4. The most likely diagnosis is:

 

1) Parosteal osteosarcoma

3) Periosteal chondroma

2) Periosteal osteosarcoma

5) Heterotopic ossification

4) Osteochondroma

 

The anteroposterior radiograph shows a lesion projecting from the medial aspect of the distal femur. The magnetic resonance imaging scan shows the cortices of the lesion are continuous with the underlying femur, and the medullary cavity of the lesion is continous with the medullary cavity of the distal femur. These findings are consistent with an osteochondroma. The bony stalk of the osteochondroma is covered by a cap of cartilage. The cartilage cap is low signal on T-weighted images and high signal on T2 weighted imagesa. The thickness of the cartilage cap is approximately 8 mm. There is no evidence of malignant change.

 

The radiographic findings of an osteochondroma are:

 

 

The underlying bone and the osteochondroma have continuous cortices. The medullary cavity of the bone and osteochondroma are continuous.

 

The lesion may have a narrow stalk (pedunculated) or a broad base (sessile).

 

Secondary chondrosarcomas may develop in an osteochondroma. The radiographic features are: Destruction of the underlying subchondral cortex of the osteochondroma

 

Lytic destruction of the cartilage cap

 

Formation of a soft tissue mass (often calcified)

 

Magnetic resonance image shows a markedly thickened cap (larger than 1 cm in adults) This patient has an osteochondroma.

Correct Answer: Osteochondroma

 

 

Slide 1 Slide 2 Slide 3 Slide 4

A 15-year-old boy has a hard mass over his distal femur. He has pain when playing sporting activities. His plain radiographs are shown in Slides 1 and 2. The axial T1- and T2-weighted images are shown in Slides 3 and 4. The most appropriate treatment is:

 

1) Needle biopsy

3) Removal of the mass

2) Incisional biopsy

5) Further evaluation with a computerized tomography scan

4) Further evaluation with a technetium bone scan

 

The anteroposterior radiograph shows a lesion projecting from the medial aspect of the distal femur. The magnetic resonance imaging scan shows the cortices of the lesion are continuous with the underlying femur, and the medullary cavity of the lesion is continous with the medullary cavity of the distal femur. These findings are consistent with an osteochondroma. The bony stalk of the osteochondroma is covered by a cap of cartilage. The cartilage cap is low signal on T2-weighted images. The thickness of the cartilage cap is approximately 8 mm. There is no evidence of malignant change.

 

The radiographic findings of an osteochondroma are:

 

 

The underlying bone and the osteochondroma have continuous cortices. The medullary cavity of the bone and osteochondroma are continuous.

 

The lesion may have a narrow stalk (pedunculated) or a broad base (sessile).

 

Secondary chondrosarcomas may develop in an osteochondroma. The radiographic features are: Destruction of the underlying subchondral cortex of the osteochondroma

 

Lytic destruction of the cartilage cap

 

Formation of a soft tissue mass (often calcified)

 

Magnetic resonance image shows a markedly thickened cap (larger than 1 cm in adults)

This patient has an osteochondroma. The most appropriate treatment is excision because the patient is symptomatic. Biopsy is not necessary prior to treatment and no further imaging is required.

Correct Answer: Removal of the mass

 

 

Slide 1 Slide 2 Slide 3 Slide 4

 

 

 

Slide 5 Slide 6

A 15-year-old boy has a hard mass over his distal femur. He has pain when playing sporting activities. His plain radiographs are shown in Slides 1 and 2. The axial T1- and T2-weighted images are shown in Slides 3 and 4. The mass is removed and the biopsy specimens are shown in Slides 5 and 6. The most likely diagnosis is:

 

1) Parosteal osteosarcoma

3) Periosteal chondroma

2) Periosteal osteosarcoma

5) Osteochondroma

4) Periosteal chondrosarcoma

 

The anteroposterior radiograph shows a lesion projecting from the medial aspect of the distal femur. The magnetic resonance imaging scan shows the cortices of the lesion are continuous with the underlying femur, and the medullary cavity of the lesion is continous with the medullary cavity of the distal femur. These findings are consistent with an osteochondroma. The bony stalk of the osteochondroma is covered by a cap of cartilage. The cartilage cap is low signal on T2-weighted images. The thickness of the cartilage cap is approximately 8 mm. There is no evidence of malignant change.

 

The radiographic findings of an osteochondroma are:

 

 

The underlying bone and the osteochondroma have continuous cortices. The medullary cavity of the bone and osteochondroma are continuous.

 

The lesion may have a narrow stalk (pedunculated) or a broad base (sessile).

 

Secondary chondrosarcomas may develop in an osteochondroma. The radiographic features are: Destruction of the underlying subchondral cortex of the osteochondroma

 

Lytic destruction of the cartilage cap

 

Formation of a soft tissue mass (often calcified)

 

Magnetic resonance image shows a markedly thickened cap (larger than 1 cm in adults)

This patient has an osteochondroma. The biopsy specimen shows a lesion with a thin cartilage cap. The high power view shows the typical features of an osteochondroma with a linear arrangement of the chondrocytes.

Correct Answer: Osteochondroma

 

 

Slide 1 Slide 2 Slide 3 Slide 4

A 15-year-old boy has a hard mass over his distal femur. He has pain when playing sporting activities. His plain radiographs are shown in Slides 1 and 2. The axial T1- and T2-weighted images are shown in Slides 3 and 4. What is the stage of this lesion according to the system of the Musculoskletal Tumor Society:

 

1) Stage 1

3) Stage 3

2) Stage 2

5) Stage II

4) Stage I

 

The anteroposterior radiograph shows a lesion projecting from the medial aspect of the distal femur. The magnetic resonance imaging scan shows the cortices of the lesion are continuous with the underlying femur, and the medullary cavity of the lesion is continous with the medullary cavity of the distal femur. These findings are consistent with an osteochondroma. The bony stalk of the osteochondroma is covered by a cap of cartilage. The cartilage cap is low signal on T2-weighted images. The thickness of the cartilage cap is approximately 8 mm. There is no evidence of malignant change.

 

The radiographic findings of an osteochondroma are:

 

 

The underlying bone and the osteochondroma have continuous cortices. The medullary cavity of the bone and osteochondroma are continuous.

 

The lesion may have a narrow stalk (pedunculated) or a broad base (sessile).

 

Secondary chondrosarcomas may develop in an osteochondroma. The radiographic features are: Destruction of the underlying subchondral cortex of the osteochondroma

 

Lytic destruction of the cartilage cap

 

Formation of a soft tissue mass (often calcified)

 

Magnetic resonance image shows a markedly thickened cap (larger than 1 cm in adults) This patient has an osteochondroma. Because the patient is symptomatic, this lesion is Stage 2. Remember the Musculoskletal Tumor Society staging system:

Stage 1 Inactive, latent

Stage 2 Active, symptomatic

Stage 3 Aggressive Â

Stage I Low grade malignant

Stage II High grade malignant

Stage III Metastatic disease Correct Answer: Stage 2

 

Which of the following statements is false regarding liposarcomas:

 

1) Most liposarcomas occur below the fascia.

3) Liposarcomas rarely occur in children.

2) Lipomas may transform into a liposarcoma

5) Liposarcomas are not induced by irradiation.

4) Reciprocal transformation (chromosome 12 and 16) occurs in myxoid liposarcoma.

 

Liposarcomas account for approximately 10% to 15% of sarcomas. Some general statements about liposarcomas are listed below:

 

  1. Most liposarcomas occur below the fascia.

  2. Superficial liposarcomas have an excellent prognosis.

  3. Lipomas do not undergo transformation into liposarcomas.

  4. Liposarcomas rarely occur in children.

  5. Reciprocal transformation between chromosomes 12 and 16 occurs in myxoid and round cell liposarcoma.

  6. Well-differentiated liposarcomas have giant marker and ring chromosome abnormalities with amplification of MDM2, SAS, and GLI genes.

  7. Liposarcomas express peroxisome proliferator-activated receptor gamma (PPAR gamma).

  8. Liposarcomas are not induced by irradiation.

Correct Answer: Lipomas may transform into a liposarcoma

 

 

3676. (1301) Q9-1671:

Which of the following statements is false regarding liposarcomas:

 

1) Liposarcomas rarely develop from lipomas.

3) Most liposarcomas occur below the deep fascia.

2) Liposarcomas rarely occur in children.

5) Reciprocal transformation (chromosome 12 and 16) occurs in myxoid liposarcoma.

4) Well-differentiated liposarcomas never have chromosomal abnormalities.

 

Liposarcomas account for approximately 10% to 15% of sarcomas. Some general statements about liposarcomas are listed below:

 

  1. Most liposarcomas occur below the fascia.

  2. Superficial liposarcomas have an excellent prognosis.

  3. Lipomas do not undergo transformation into liposarcomas.

  4. Liposarcomas rarely occur in children.

  5. Reciprocal transformation between chromosomes 12 and 16 occurs in myxoid and round cell liposarcoma.

  6. Well-differentiated liposarcomas have giant marker and ring chromosome abnormalities with amplification of MDM2, SAS, and GLI genes.

  7. Liposarcomas express peroxisome proliferator-activated receptor gamma (PPAR gamma).

  8. Liposarcomas are not induced by irradiation.

Correct Answer: Well-differentiated liposarcomas never have chromosomal abnormalities.

 

Which of the following soft tissue lesions has a characteristic reciprocal transformation between chromosomes 12 and 16:

 

1) Well-differentiated lipoma-like liposarcoma

3) Intramuscular lipomas

2) Atypical lipoma

5) Pleomorphic liposarcoma

4) Myxoid liposarcoma

 

Liposarcomas account for approximately 10% to 15% of sarcomas. Some general statements about liposarcomas are listed below. Liposarcomas may have chromosomal abnormalities:

  1. Reciprocal transformation between chromosomes 12 and 16 occurs in myxoid and round cell liposarcoma.

  2. Well-differentiated liposarcomas have giant marker and ring chromosome abnormalities with amplification of MDM2, SAS, and GLI genes.

     

  3. Liposarcomas express peroxisome proliferator-activated receptor gamma (PPAR gamma). Correct Answer: Myxoid liposarcoma

 

3678. (1303) Q9-1673:

Myxoid liposarcomas have which of the following chromosomal alterations:

 

1) Giant marker and ring chromosomes

3) Translocation between chromosomes 11 and 22

2) Reciprocal translocation between chromosomes 12 and 16

5) Translocation between chromosomes 9 and 22

4) Reciprocal translocation of chromosomes X and 18

 

Many benign and malignant soft tissue tumors have specific chromosomal abnormalities. A few common abnormalities include:

 

Well-differentiated liposarcoma Giant marker and ring

chromosomes

Myxoid liposarcoma Translocation between chromosomes 12 and 16

Ewings/primitive neuroectodermal tumors

Translocation between chromosomes 11 and 22

Synovial sarcoma Translocation between chromosomes X and 18

Myxoid chondrosarcoma Translocation between

chromosomes 9 and 22

Correct Answer: Reciprocal translocation between chromosomes 12 and 16

 

Soft tissue Ewings or primitive neuroectodermal tumors have which of the following cytogenetic abnormalities:

 

1) Giant marker and ring chromosomes

3) Translocation between chromosomes 11 and 22

2) Reciprocal translocation between chromosomes 12 and 16

5) Translocation between chromosomes 9 and 22

4) Reciprocal translocation of chromosomes X and 18

 

Many benign and malignant soft tissue tumors have specific chromosomal abnormalities. A few common abnormalities include:

 

Well-differentiated liposarcoma Giant marker and ring

chromosomes

Myxoid liposarcoma Translocation between chromosomes 12 and 16

Ewings/primitive neuroectodermal tumors

Translocation between chromosomes 11 and 22

Synovial sarcoma Translocation between chromosomes X and 18

Myxoid chondrosarcoma Translocation between

chromosomes 9 and 22

Correct Answer: Translocation between chromosomes 11 and 22

 

 

3680. (1305) Q9-1675:

Well-differentiated lipoma-like liposarcomas have which of the following characteristic chromosomal alterations:

 

1) Giant marker and ring chromosomes

3) Translocation between chromosomes 11 and 22

2) Reciprocal translocation between chromosomes 12 and 16

5) Translocation between chromosomes 9 and 22

4) Reciprocal translocation of chromosomes X and 18

 

Many benign and malignant soft tissue tumors have specific chromosomal abnormalities. A few common abnormalities include:

 

Well-differentiated liposarcoma Giant marker and ring

chromosomes

Myxoid liposarcoma Translocation between chromosomes 12 and 16

Ewings/primitive neuroectodermal tumors

Translocation between chromosomes 11 and 22

Synovial sarcoma Translocation between chromosomes X and 18

Myxoid chondrosarcoma Translocation between

chromosomes 9 and 22

Correct Answer: Giant marker and ring chromosomes

 

Synovial sarcomas have which of the following chromosomal abnormalities:

 

1) Giant marker and ring chromosomes

3) Translocation between chromosomes 11 and 22

2) Reciprocal translocation between chromosomes 12 and 16

5) Translocation between chromosomes 9 and 22

4) Reciprocal translocation of chromosomes X and 18

 

Many benign and malignant soft tissue tumors have specific chromosomal abnormalities. A few common abnormalities include:

 

Well-differentiated liposarcoma Giant marker and ring

chromosomes

Myxoid liposarcoma Translocation between chromosomes 12 and 16

Ewings/primitive neuroectodermal tumors

Translocation between chromosomes 11 and 22

Synovial sarcoma Translocation between chromosomes X and 18

Myxoid chondrosarcoma Translocation between

chromosomes 9 and 22

Correct Answer: Reciprocal translocation of chromosomes X and 18

 

 

3682. (1307) Q9-1677:

Myxoid soft tissue chondrosarcomas have which of the following chromosomal alterations:

 

1) Giant marker and ring chromosomes

3) Translocation between chromosomes 11 and 22

2) Reciprocal translocation between chromosomes 12 and 16

5) Translocation between chromosomes 9 and 22

4) Reciprocal translocation of chromosomes X and 18

 

Many benign and malignant soft tissue tumors have specific chromosomal abnormalities. A few common abnormalities include:

 

Well-differentiated liposarcoma Giant marker and ring

chromosomes

Myxoid liposarcoma Translocation between chromosomes 12 and 16

Ewings/primitive neuroectodermal tumors

Translocation between chromosomes 11 and 22

Synovial sarcoma Translocation between chromosomes X and 18

Myxoid chondrosarcoma Translocation between

chromosomes 9 and 22

Correct Answer: Translocation between chromosomes 9 and 22

 

 

3683. (1308) Q9-1678:

Which of the following most accurately describes the etiology of nonossifying fibromas of bone:

 

1) Benign neoplasm of histiocytic cells

3) Benign neoplasm of lipoblastic origin

2) Benign neoplasm of fibroblastic origin

5) Benign inflammatory process

4) Variant of normal growth

 

Nonossifying fibromas of bone are best considered as variants of normal growth. Nonossifying fibromas are rare in children younger than 2 years old, and they commonly occur at the sites of muscle attachment.Correct Answer: Variant of normal growth

 

Which of the following are the most common locations of nonossifying fibromas:

 

1) Femur and humerus

3) Tibia and fibula

2) Femur and tibia

5) Scapula and small bones of the hands and feet

4) Femur and scapula

 

Nonossifying fibromas most commonly arise in the metaphysis of long bones in close proximity to the physis. As a child grows, the lesion moves away from the physis and extends into the diaphysis. Extension into the epiphysis is rare. More than 55% of nonossifying fibromas occur about the knee. They usually arise from the posterior cortex.

 

The approximate distribution of nonossifying fibromas is: Tibia â 43%

 

Femur â 38%

 

Fibula â 8%

 

Humerus â 5%

Note:

Nonossifying fibromas rarely occur in the radius, ulna, ribs, innominate bone, clavicle, skull, mandible, scapula, and small bones of hand and wrist.

Correct Answer: Femur and tibia

 

 

3685. (1310) Q9-1680:

Which of the following describes the radiographic appearance of nonossifying fibromas:

 

1) Eccentric surface lesion eroding the external cortical surface

3) Small, round lucent lesion with surrounding sclerosis

2) Eccentric, lucent, metaphyseal lesion with sclerotic rim and overlying thinned cortex

5) Radiolucent lesion in proximal medial tibial cortex with tibia vara

4) Saucer-like radiolucent lesion within the cortex with adjacent sclerosis and periostitis Nonossifying fibromas have the following radiographic features:

 

Eccentric

 

 

Metaphyseal Sclerotic rim

 

 

Overlying cortex is thinned Lesion is based on the cortex

The other descriptions pertain to other bone tumors:

Periosteal chondroma â eccentric lesion eroding the external cortical surface Osteoid osteoma â small, round lucent lesion with surrounding sclerosis

 

Cortical desmoid â saucer-like radiolucent lesion within the cortex with adjacent sclerosis and periostitis Focal fibrocartilaginous dysplasia â radiolucent lesion in proximal medial tibial cortex with tibia vara Correct Answer: Eccentric, lucent, metaphyseal lesion with sclerotic rim and overlying thinned cortex

Which of the following describes the histologic features of nonossifying fibromas:

 

1) Multinucleated giant cells uniformly distributed in a field of mononuclear cells

3) A thin fibrous membrane with scattered inflammatory cells

2) Scattered multinucleated giants cells in a field of mononuclear cells with chondroid areas

5) Pleomorphic stroma with osteoid production

4) Storiform pattern of benign fibroblasts and scattered multinucleated giant cells Nonossifying fibromas have a characteristic histologic appearance:

 

Uniform, benign-appearing fibroblasts in a storiform pattern

 

 

 

 

Scattered multinucleated giant cells Foam cells in 30% to 50% Occasionally hemosiderin Occasionally cholesterol clefts

 

The other descriptions pertain to these lesions: Giant cell tumor â multinucleated giant cells uniformly distributed in a field of mononuclear cells Chondroblastoma â scattered multinucleated giants cells in a field of mononuclear cells with chondroid areas Unicameral bone cyst â a thin fibrous membrane with scattered inflammatory cells Osteosarcoma â pleomorphic stroma with osteoid productionCorrect Answer: Storiform pattern of benign fibroblasts and scattered multinucleated giant cells

 

 

3687. (1312) Q9-1682:

Which of the following statements is true regarding nonossifying fibromas:

 

1) Most lesions progress in size and require curettage and grafting.

3) Lesions that are greater than 50% the width of the bone and greater than 33 mm in length may be prone to fracture.

2) Aspiration and steroid injection result in rapid resolution.

5) The lesion often begins in the first year of life.

4) The risk of malignancy is related to size and the radiographic margination.

 

Nonossifying fibromas generally resolve as children become adults. The lesions involute and mineralize. Seldom are nonossifying fibromas present in an adult. The majority of patients can be treated nonoperatively. In a study of 23 patients with fractures, Areata and Peterson found that lesions that occupy less than 50% the width of the bone and are less than 33 mm in length are less prone to fracture.

 

Other important points to remember:

 

The risk of malignancy is almost nonexistent.

 

Nonossifying fibromas are never found in children younger than 2 years of age.

Correct Answer: Lesions that are greater than 50% the width of the bone and greater than 33 mm in length may be prone to fracture.

 

 

3688. (1313) Q9-1683:

Which of the following conditions has been associated with nonossifying fibromas in a small number of patients:

 

1) Primary hyperparathyroidism

3) Ventricular septal defect

2) Polycystic kidneys

5) Clubfoot and vertical talus

4) Hypophosphatemic vitamin D refractory rickets and osteomalacia

 

Nonossifying fibroma is a common benign bone tumor that may rarely be associated with hypophosphatemic vitamin D refractory rickets and osteomalacia. Once the nonossifying fibroma is removed, the rickets resolve.Correct Answer: Hypophosphatemic vitamin D refractory rickets and osteomalacia

 

Multiple large and symmetric nonossifying fibromas may occur in Jaffe-Campanacci syndrome. Which of the following also occurs in this condition:

 

1) Café-au-lait spots, mental retardation, hypogonadism, and cardiovascular abnormalities

3) Spina bifida and myelomeningocele

2) Radial club hand and atlantoaxial instability

5) Soft tissue angiomas and limb shortening with angular deformity

4) Hypophosphatemic vitamin D refractory rickets and osteomalacia The Jaffe-Campanacci syndrome consists of:

 

Multiple nonossifying fibromas that tend to be large and symmetric. The nonossifying fibromas involve the metaphyses and

 

diaphyses and may extend into the epiphysis. Café-au-lait spots

 

 

 

Mental retardation Hypogonadism Ocular abnormalities

 

Cardiovascular abnormalities

There should be at least three nonossifying fibromas. Patients who are otherwise normal and have multiple nonossifying fibromas do not have Jaffe-Campanacci syndrome.

Correct Answer: Café-au-lait spots, mental retardation, hypogonadism, and cardiovascular abnormalities

 

 

3690. (1315) Q9-1685:

Which of the following describes the probable inheritance pattern of Pagetâs disease:

 

1) Spontaneous mutation in all patients

3) Autosomal dominant

2) Autosomal recessive

5) X-linked dominant

4) X-linked recessive

 

The inheritance pattern of Pagetâs disease is most likely autosomal dominant in pattern. A first degree relative has a seven times greater chance of developing Pagetâs disease than the general population. The exact inheritance is unknown although the human leucocyte antigen (HLA) locus and chromosome 18q (10,11) have been implicated.Correct Answer: Autosomal dominant

 

 

3691. (1316) Q9-1686:

In which of the following countries is Pagetâs disease most rare:

 

1) Europe and North America

3) France, Spain, and West Germany

2) Australia and New Zealand

5) China, India, and Malaysia

4) Italy and Greece

 

Pagetâs disease is common in all countries except China, India, Malaysia, and sub-Saharan Africa. Pagetâs disease is common in people older than 55 years in the following countries:

 

Europe

 

 

North America England (4.6%)

 

France (2.4%)

 

Ireland (0.7 to 1.7%)

 

Spain (1.3%)

 

West Germany (1.3%)

 

Italy (0.5%)

 

Greece (0.5%)

Correct Answer: China, India, and Malaysia

 

Which of the following infections has been linked to the development of Pagetâs disease:

 

1) Bacterial infection with spirochetes

3) Viral infection with human immunodeficiency virus

2) Viral infection with a paramyxovirus

5) Bacterial infection with Yersinia

4) Bacterial infection with streptococci

 

Infection with a paramyxovirus has been implicated in Pagetâs disease. Inclusions that resemble viral nucleocapsid have been found with electron microscopy in the nuclei and cytoplasm of osteoclasts in Pagetâs disease. It is still controversial whether the virus belongs to the measles, canine distemper, or respiratory syncytial viruses.Correct Answer: Viral infection with a paramyxovirus

 

 

3693. (1318) Q9-1688:

Which of the following is not elevated in patients with active Pagetâs disease:

 

1) Urinary total hydroxyproline

3) Serum calcium

2) Serum alkaline phosphatase

5) Urinary deoxypyridinoline

4) Urinary N-telopeptide

 

In Pagetâs disease, serum and urinary markers reflect the rate of increased bone turnover (bone resorption and bone formation): Elevated in active Pagetâs disease:

 

Serum alkaline phosphatase

 

 

Serum bone-specific alkaline phosphatase Urinary hydroxyproline

 

 

Urinary N-telopeptide of collagen Urinary deoxypyridinoline

The serum calcium level is usually normal in Pagetâs disease. However, if a patient has active and extensive Pagetâs disease and the individual is immobilized, then the serum calcium level may become elevated.

Correct Answer: Serum calcium

 

 

3694. (1319) Q9-1689:

Which of the following bones are rarely affected by Pagetâs disease:

 

1) Pelvis and femur

3) Femur and tibia

2) Skull and spine

5) Hands and feet

4) Ribs and facial bones

 

The bones most commonly involved in Pagetâs disease are the pelvis, femur, spine, skull, and tibia. Less commonly involved bones are the clavicles, scapulae, ribs, and facial bones.

 

Involvement of the hands and feet is rare in Pagetâs disease. Correct Answer: Hands and feet

Which of the following tests or procedures is most commonly used to establish the diagnosis of Pagetâs disease:

 

1) Technetium bone scan

3) Urinary hydroxyproline

2) Serum alkaline phosphatase

5) Plain radiographs

4) Bone biopsy

 

Plain radiographs are commonly used to diagnose Pagetâs disease. The radiographic features include: Enlargement and expansion of the bone

 

 

Coarsened trabeculae Cortical thickening

 

Lucent and sclerotic changes

Technetium bone scans are sensitive to sites of involvement. In general, at the time of diagnosis, there will not be a change in the number of the involved bones.

Serum alkaline phosphatase level is elevated in approximately 95% of patients. Urinary markers of bone turnovers will also be elevated and include hyrodroxyproline, N-telopeptide, and deoxypyridinoline.

 

Bone biopsy is seldom required to diagnose Pagetâs disease. Correct Answer: Plain radiographs

 

3696. (1321) Q9-1691:

Which of the following biphosphonates is approved for use as an antipagetic medication and may halt both bone resorption by osteoclasts and bone formation by osteoblasts:

 

1) Etidronate

3) Alendronate

2) Pamidronate

5) Risedronate

4) Tiludronate

 

Etidronate was the first biphosphonate used in the treatment of Pagetâs disease. It stopped both osteoclastic resorption and bone formation. Etidronate could only be used for periods of 6 months because prolonged etidronate therapy caused osteomalacia.

 

The newer diphosphonates are more potent than etidronate and do not inhibit bone formation. Correct Answer: Etidronate

 

Slide 1 Slide 2

A 38-year-old man has a 4-month history of increasing right hip pain. The plain radiographs are shown in Slide 1 and a biopsy specimen in Slide 2. The most likely diagnosis is:

 

1) Osteochondroma

3) High grade surface osteosarcoma

2) Periosteal chondroma

5) Parosteal osteosarcoma

4) Periosteal chondrosarcoma

 

This patient has a large lesion arising from the surface of the bone. There is matrix mineralization of the lesion that shows rings and stipples. This matrix pattern is consistent with a cartilage lesion. The biopsy specimen shows the blue matrix of cartilage and abundant clumps of chondrocytes. This patient has a surface chondrosarcoma. Juxtacortical chondrosarcoma and periosteal chondrosarcoma are names that can be used interchangeably.Correct Answer: Periosteal chondrosarcoma

 

 

3698. (1323) Q9-1693:

 

 

 

Slide 1 Slide 2

A 38-year-old man has a 4-month history of increasing right hip pain. The plain radiographs are shown in Slide 1 and a biopsy specimen in Slide 2. The most appropriate treatment is:

 

1) Observation

3) Chemotherapy and external beam irradiation

2) Preoperative chemotherapy followed by surgical resection

5) Wide surgical resection and reconstruction

4) External beam irradiation alone

 

This patient has a large lesion arising from the surface of the bone. There is matrix mineralization of the lesion that shows rings and stipples. This matrix pattern is consistent with a cartilage lesion. The biopsy specimen shows the blue matrix of cartilage and abundant clumps of chondrocytes. This patient has a surface chondrosarcoma. Juxtacortical chondrosarcoma and periosteal chondrosarcoma are names that can be used interchangeably.

 

The most appropriate treatment is wide surgical resection and reconstruction. The reconstruction can be performed with a custom prosthesis or an allograft prosthetic composite.

Correct Answer: Wide surgical resection and reconstruction

 

 

Slide 1 Slide 2

A 38-year-old man has a 4-month history of increasing right hip pain. The plain radiographs are shown in Slide 1 and a biopsy specimen in Slide 2. The most likely cause of his hip pain is:

 

1) A post-traumatic condition

3) A benign neoplasm

2) An infectious process

5) A developmental condition

4) A malignant neoplasm

 

This patient has a large lesion arising from the surface of the bone. There is matrix mineralization of the lesion that shows rings and stipples. This matrix pattern is consistent with a cartilage lesion. The biopsy specimen shows the blue matrix of cartilage and abundant clumps of chondrocytes. This patient has a surface chondrosarcoma. Juxtacortical chondrosarcoma and periosteal chondrosarcoma are names that can be used interchangeably.

 

This patient has a malignant bone condition. This particular radiographic appearance can be misinterpreted as a benign condition: Posttraumatic condition: Heterotopic ossification

 

 

Infectious process: Osteomyelitis Benign neoplasm: Osteochondroma

Correct Answer: A malignant neoplasm

 

 

3700. (1325) Q9-1695:

Which of the following is a common finding in patients with tumoral calcinosis:

 

1) Hypercalcemia

3) Hyperphosphatemia

2) Elevated alkaline phosphatase

5) Hypercalciuric

4) Hypocalcemia

 

Tumoral calcinosis is a heritable condition that is characterized by periarticular metastatic calcification. Most patients are black, and the inheritance is usually autosomal recessive. Metastatic calcifications occur around joints and in the skin, marrow, teeth, and blood vessels. The periarticular masses may grow quite large and are attached to the fascia, but they are extra-articular. The masses may occur at the shoulder, hip, and elbow.

 

 

 

Radiographically: The masses are composed of heavy, amorphous calcification in nodules. Laboratory:

  1. Serum calcium normal

  2. Serum alkaline phosphatase normal

  3. Urine calcium is low

     

  4. Serum phosphate is often high When symptomatic, the masses are excised. Correct Answer: Hyperphosphatemia

Which of the following statements is false concerning tumoral calcinosis:

 

1) Most patients are white.

3) The periarticular masses are extracapsular.

2) Serum calcium levels are usually normal.

5) The most common inheritance is autosomal recessive.

4) Serum phosphate levels are high.

 

Tumoral calcinosis is a heritable condition that is characterized by periarticular metastatic calcification. Most patients are black, and the inheritance is usually autosomal recessive. Metastatic calcifications occur around joints and in the skin, marrow, teeth, and blood vessels. The periarticular masses may grow quite large and are attached to the fascia, but they are extra-articular. The masses may occur at the shoulder, hip, and elbow.

 

 

 

Radiographically: The masses are composed of heavy, amorphous calcification in nodules. Laboratory:

  1. Serum calcium normal

  2. Serum alkaline phosphatase normal

  3. Urine calcium is low

     

  4. Serum phosphate is often high When symptomatic, the masses are excised. Correct Answer: Most patients are white.

 

3702. (1327) Q9-1697:

Which of the following statements is false concerning tumoral calcinosis:

 

1) Most patients are black.

3) The periarticular masses are intracapsular and restrict motion.

2) Serum calcium levels are usually normal.

5) The most common inheritance is autosomal recessive.

4) Serum phosphate levels are high.

 

Tumoral calcinosis is a heritable condition that is characterized by periarticular metastatic calcification. Most patients are black, and the inheritance is usually autosomal recessive. Metastatic calcifications occur around joints and in the skin, marrow, teeth, and blood vessels. The periarticular masses may grow quite large and are attached to the fascia, but they are extra-articular. The masses may occur at the shoulder, hip, and elbow.

 

 

 

Radiographically: The masses are composed of heavy, amorphous calcification in nodules. Laboratory:

  1. Serum calcium normal

  2. Serum alkaline phosphatase normal

  3. Urine calcium is low

  4. Serum phosphate is often high When symptomatic, the masses are excised.

Correct Answer: The periarticular masses are intracapsular and restrict motion.

 

Which of the following describes the histopathology of tumoral calcinosis:

 

1) Malignant stromal cells with direct osteoid production

3) Foreign-body granuloma reactions forming multilocular, cystic structures

2) Lymphocytes between edematous muscle fibers and angiogenic proliferative tissue

5) Zonal pattern with mature bone at periphery and immature tissue at periphery

4) Biphasic with epithelial and fibrous cells

 

Tumoral calcinosis is a heritable condition that is characterized by periarticular metastatic calcification. Most patients are black, and the inheritance is usually autosomal recessive. Metastatic calcifications occur around joints and in the skin, marrow, teeth, and blood vessels. The periarticular masses may grow quite large and are attached to the fascia, but they are extra-articular. The masses may occur at the shoulder, hip, and elbow.

 

The histologic features are:

 

 

Foreign-body granuloma that react with multilocular, cystic structures Cysts with tough connective tissue walls

 

Mature lesions filled with calcareous material (viscous milky fluid) The other answers refer to:

 

Malignant stromal cells with direct osteoid production: osteosarcoma

 

Lymphocytes between edematous muscle fibers and angiogenic proliferative tissue: fibrodysplasia (myositis) ossificans progressiva

 

Biphasic with epithelial and fibrous cells: synovial sarcoma

 

Zonal pattern with mature bone at periphery and immature tissue at periphery: heterotopic ossification (myositis ossificans)

Correct Answer: Foreign-body granuloma reactions forming multilocular, cystic structures

 

 

3704. (1329) Q9-1699:

The treatment of tumoral calcinosis is:

 

1) Excision with an intralesional or marginal margin

3) Wide excision and external beam irradiation

2) Wide excision with a cuff of normal muscle

5) Preoperative external beam irradiation followed with wide excision

4) Preoperative chemotherapy and wide excision

 

Tumoral calcinosis is a heritable condition that is characterized by periarticular metastatic calcification. Most patients are black, and the inheritance is usually autosomal recessive. Metastatic calcifications occur around joints and in the skin, marrow, teeth, and blood vessels. The periarticular masses may grow quite large and are attached to the fascia, but they are extra-articular. The masses may occur at the shoulder, hip, and elbow.

 

 

 

Radiographically: The masses are composed of heavy, amorphous calcification in nodules. Laboratory:

  1. Serum calcium normal

  2. Serum alkaline phosphatase normal

  3. Urine calcium is low

  4. Serum phosphate is often high

When symptomatic, the masses are excised. This lesion is benign and removal of normal tissue and radiation therapy are not necessary.

Correct Answer: Excision with an intralesional or marginal margin

 

 

Slide 1 Slide 2

A 20-year-old woman has a large mass over the right hip. An anteroposterior and oblique radiographs are shown in Slides 1 and

2. The most likely diagnosis is:

 

1) Parosteal osteosarcoma

3) Synovial sarcoma

2) Periosteal osteosarcoma

5) Fibrodysplasia ossificans progressiva

4) Tumoral calcinosis

 

Tumoral calcinosis is a heritable condition that is characterized by periarticular metastatic calcification. Most patients are black, and the inheritance is usually autosomal recessive. Metastatic calcifications occur around joints and in the skin, marrow, teeth, and blood vessels. The periarticular masses may grow quite large and are attached to the fascia, but they are extra-articular. The masses may occur at the shoulder, hip, and elbow.

 

 

 

Radiographically: The masses are composed of heavy, amorphous calcification in nodules. Laboratory:

  1. Serum calcium normal

  2. Serum alkaline phosphatase normal

  3. Urine calcium is low

  4. Serum phosphate is often high

When symptomatic, the masses are excised. This lesion is benign and removal of normal tissue and radiation therapy are not necessary.

Notice the lobules of amorphous calcification that are centered about the hip joint. This is the characteristic appearance of tumoral calcinosis.

Correct Answer: Tumoral calcinosis

 

 

Slide 1 Slide 2

A 20-year-old patient who is otherwise healthy has a soft mass over her hip. The mass has formed over the past 2 years. The radiographs are shown in Slides 1 and 2. Which of the following would be a common finding:

 

1) Hypercalcemia

3) Hyperphosphatemia

2) Elevated alkaline phosphatase

5) Hypercalciuric

4) Hypocalcemia

 

Notice the lobules of amorphous calcification that are centered about the hip joint. This is the characteristic appearance of tumoral calcinosis.

Tumoral calcinosis is a heritable condition that is characterized by periarticular metastatic calcification. Most patients are black, and the inheritance is usually autosomal recessive. Metastatic calcifications occur around joints and in the skin, marrow, teeth, and blood vessels. The periarticular masses may grow quite large and are attached to the fascia, but they are extra-articular. The masses may occur at the shoulder, hip, and elbow.

 

 

 

Radiographically: The masses are composed of heavy, amorphous calcification in nodules. Laboratory:

  1. Serum calcium normal

  2. Serum alkaline phosphatase normal

  3. Urine calcium is low

     

  4. Serum phosphate is often high Correct Answer: Hyperphosphatemia

Slide 1 Slide 2 Slide 3 Slide 4

Slide 5

A 50-year-old woman has a 1-year history of moderate to severe pain in the tibia. Her plain radiographs are shown in Slide 1,

and the axial and sagittal T1-weighted images in Slides 2 and 3. Low- and high-power hematoxylin and eosin biopsy specimens are shown in Slides 4 and 5. The most likely diagnosis based upon the imaging studies and biopsy specimen is:

 

1) Metastatic bone disease

3) Malignant fibrous histiocytoma

2) Multiple myeloma

5) Chondrosarcoma

4) Lymphoma

 

The lateral radiograph shows bone destruction and formation over a long segment of the tibia. There is cortical thickening and areas of bone destruction. The T1-weighted images show replacement of the marrow with low signal intensity. The normal marrow is high signal (white) and the tumor is low signal (dark). The biopsy shows a mixture of large and small blue cells. This is the histologic appearance of lymphoma.

 

Note that the other choices have characteristic histologic appearances:

 

Metastatic bone disease: epithelial cells organized in clusters in a fibrous background

 

Multiple myeloma: sheets of plasma cells with an eccentric nucleus with peripheral chromatin. There is also perinuclear clearing, or a halo, where the Golgi apparatus is located.

 

Chondrosarcoma: atypical chondrocytes in a blue chondroid matrix

 

 

Malignant fibrous histiocytoma: fascicles of fibroblasts and histiocytes in a storiform pattern Correct Answer: Lymphoma

Slide 1 Slide 2 Slide 3 Slide 4

Slide 5

A 50-year-old woman has a 1-year history of moderate to severe pain in the tibia. Her plain radiographs are shown in Slide 1,

and the axial and sagittal T1-weighted images in Slides 2 and 3. Low- and high-power hematoxylin and eosin biopsy specimens are shown in Slides 4 and 5. Which of the following would be the most appropriate treatment:

 

1) Above the knee amputation

3) Preoperative chemotherapy followed by resection/reconstruction

2) Wide resection and custom tibial knee replacement

5) Curettage and cement augmentation

4) Chemotherapy and external beam irradiation

 

The lateral radiograph shows bone destruction and formation over a long segment of the tibia. There is cortical thickening and areas of bone destruction. The T1-weighted images show replacement of the marrow with low signal intensity. The normal marrow is high signal (white) and the tumor is low signal (dark). The biopsy shows a mixture of large and small blue cells. This is the histologic appearance of lymphoma.

 

Note that the other choices have characteristic histologic appearances:

 

Metastatic bone disease: epithelial cells organized in clusters in a fibrous background

 

Multiple myeloma: sheets of plasma cells with an eccentric nucleus with peripheral chromatin. There is also perinuclear clearing, or a halo, where the Golgi apparatus is located.

 

Chondrosarcoma: atypical chondrocytes in a blue chondroid matrix

 

Malignant fibrous histiocytoma: fascicles of fibroblasts and histiocytes in a storiform pattern

The treatment of lymphoma of bone in the adult is generally chemotherapy and external beam irradiation. Surgery is usually reserved for fractures.

Correct Answer: Chemotherapy and external beam irradiation

 

Which of the following tumors affects young adults and commonly occurs in the fingers, hands, and forearms:

 

1) Synovial sarcoma

3) Malignant fibrous histiocytoma

2) Liposarcoma

5) Epithelioid sarcoma

4) Fibrosarcoma

 

Epithelioid sarcoma is a morphologically distinctive neoplasm that is commonly confused with other conditions such as granulomas, granuloma annulare, ulcerating squamous cell carcinoma, and synovial sarcoma. Epithelioid sarcoma occurs in young people (10-35 years old, median age 26). The most common locations include the fingers, hands, and forearms.

 

Sites of epithelioid sarcoma: Fingers, hands, and forearms

 

 

Knee and lower leg (especially pretibial area) Buttocks and thigh

 

 

Shoulder and arm Ankle, foot, and toe

Correct Answer: Epithelioid sarcoma

 

 

3710. (1335) Q9-1705:

Which of the following describes a common clinical presentation of an epithelioid sarcoma:

 

1) A large mass in the posterior thigh

3) A soft, subcutaneous mass

2) A large mass adjacent to the bursal knee structures

5) A large mass on the lateral border of the foot

4) An ulcerated nodule on the hand

 

Epithelioid sarcoma is a morphologically distinctive neoplasm that is commonly confused with other conditions such as granulomas, granuloma annulare, ulcerating squamous cell carcinoma, and synovial sarcoma. Epithelioid sarcoma occurs in young people (10-35 years old, median age 26). The most common locations include the fingers, hands, and forearms.

 

A common clinical presentation is an ulcerated nodule on the hand. Sites of epithelioid sarcoma:

 

Fingers, hands, and forearms

 

 

Knee and lower leg (especially pretibial area) Buttocks and thigh

 

 

Shoulder and arm Ankle, foot, and toe

The other answers describe common clinical presentations of several tumors:

 

 

 

Malignant fibrous histiocytoma â A large mass in the posterior thigh Synovial sarcoma â A large mass adjacent to the bursal knee structures Lipoma â A soft, subcutaneous mass

 

 

Synovial sarcoma â A soft, subcutaneous mass Correct Answer: An ulcerated nodule on the hand

Which of the following describes the histologic appearance of epithelioid sarcoma:

 

1) Storiform pattern of pleomorphic spindle cells and histiocytes

3) Epithelial cells and spindle cells

2) Herringbone pattern of spindle cells

5) Small blue cells in an alveolar pattern

4) Nodules with central necrosis and ovoid or polygonal cells with eosinophilic cytoplasm There are several histologic features of epithelioid sarcoma:

 

Nodular growth with central necrosis

 

Ovoid or polygonal cells with deeply eosinophilic cytoplasm

Histologically, the cells may lack pleomorphism and may be confused with granulomas, granuloma annulare, squamous cell carcinoma, necrotizing infectious granuloma, necrobiosis lipoidica, and rheumatoid nodules.

Correct Answer: Nodules with central necrosis and ovoid or polygonal cells with eosinophilic cytoplasm

 

 

3712. (1337) Q9-1707:

Which of the following forms of rhabdomyosarcoma has a poor prognosis:

 

1) Botryoid

3) Embryonal

2) Spindle cell

5) Rhabdoid

4) Alveolar

 

Alveolar rhabdomyosarcoma carries the worse prognosis. Superior prognosis

Botryoid rhabdomyosarcoma Spindle cell rhabdomyosarcoma

Intermediate prognosis

Embryonal rhabdomyosarcoma Unevaluable prognosis

 

Rhabdomyosarcoma with rhabdoid features Correct Answer: Alveolar

 

3713. (1338) Q9-1708:

Which of the following is the most common soft tissue tumor of infancy and childhood:

 

1) Rhabdomyosarcoma

3) Nodular fasciitis

2) Hemangioma

5) Synovial sarcoma

4) Tendon sheath fibroma

 

Hemangiomas are the most common soft tissue tumors in infancy and childhood.

Rhabdomyosarcoma is the most common sarcoma in children and synovial sarcoma is the most common soft tissue sarcoma of the foot and ankle.

Correct Answer: Hemangioma

 

Which of the following is a typical radiographic finding in intramuscular hemangiomas:

 

1) Nodules of amorphous calcification

3) Small round phleboliths

2) Dense, cloudy mineralization

5) Rounded or ovoid radiolucency

4) Nodules with rings and stipples

 

Hemangiomas may show small round phleboliths. These phleboliths are similar in appearance to those found in the pelvic veins. The other answers describe specific entities:

 

 

 

Tumoral calcinosis â Nodules of amorphous calcification Osteosarcoma â Dense, cloudy mineralization Chondromas â Nodules with rings and stipples

 

 

Deep lipoma â Rounded or oval radiolucency Correct Answer: Small round phleboliths

 

3715. (1340) Q9-1710:

A 15-year-old girl has a 2-year history of atraumatic, recurrent episodes of pain, swelling, and joint effusions. Based upon the history, the most likely diagnosis is:

 

1) Synovial sarcoma

3) Synovial hemangioma

2) Osteonecrosis

5) Rhabdomyosarcoma

4) Fibroma of tendon sheath

 

Synovial hemangiomas are an uncommon cause of joint pain in children. When they do occur, the presentation is atraumatic, recurrent episodes of pain, swelling, and joint effusions.

Pathologically:

 

Synovial hemangiomas are cavernous hemangiomas. The vessels are separated by an edematous, myxoid, or focally hyalinized matrix.

 

The synovium has a villous configuration over the hemangioma and the synovium has large amounts of hemosiderin pigment.

 

Treatment of synovial hemangiomas is excision. Correct Answer: Synovial hemangioma

 

3716. (1341) Q9-1711:

Which of the following has the highest risk of recurrence following surgical excision:

 

1) Nodular fasciitis

3) Intramuscular lipoma

2) Schwannoma

5) Angiolipoma

4) Intramuscular hemangioma

 

Intramuscular hemangiomas have an extremely high local failure rate, 20% to >50%. Surgery must be avoided if possible. Symptomatic treatment includes nonsteroidal anti-inflammatory drugs, compression stockings, and activity modification.

Embolization or sclerosing therapy is the mainstay of current treatment.Correct Answer: Intramuscular hemangioma

 

Which of the following is the most common soft tissue sarcoma in adults:

 

1) Liposarcoma

3) Synovial sarcoma

2) Malignant fibrous histiocytoma

5) Clear cell sarcoma

4) Epithelioid sarcoma

 

Malignant fibrous histiocytoma is the most common soft tissue sarcoma in adults, followed by liposarcoma.Correct Answer: Malignant fibrous histiocytoma

 

 

3718. (1343) Q9-1713:

Which of the following is the most common form of malignant fibrous histiocytoma:

 

1) Storiform-pleomorphic

3) Giant cell

2) Myxoid

5) Angiomatoid fibrous histiocytoma

4) Inflammatory

 

The storiform-pleomorphic type is the most common form of malignant fibrous histiocytoma.

 

Myxoid: Second most common, accounts for 25%

 

 

Giant cell: Contains numerous osteoclast-like giant cells Inflammatory: Most common in the retroperitoneum

Correct Answer: Storiform-pleomorphic

 

 

3719. (1344) Q9-1714:

Which of the following is the most common location for malignant fibrous histiocytoma:

 

1) Hand

3) Retroperitoneum

2) Foot and ankle

5) Chest wall

4) Thigh

 

The most common location of malignant fibrous histiocytoma is the thigh. Remember these common locations for the following tumors:

 

 

Synovial sarcoma â Foot/ankle Epithelioid sarcoma â Hand

 

 

Melanoma â Plantar surface of the foot Correct Answer: Thigh

Which of the following describes the histologic features of malignant fibrous histiocytoma:

 

1) Fascicles of spindle cells in a herringbone pattern

3) Epithelial cells in a gland-like pattern and spindle cells

2) Spindle cells in short fascicles in a storiform pattern around slit-like vessels

5) Small blue cells in an alveolar pattern

4) Nodules of epithelial cells with central necrosis

 

The histologic pattern of malignant fibrous histiocytoma includes:

 

 

Spindle cells in short fascicles in a storiform, or cartwheel, pattern arranged around slit-like vessels. Pleomorphic areas that contain plump fibroblasts and rounded histiocytes.

 

A large number of giant cells with hyperchromatic nuclei.

The other answers describe the following soft tissue tumors:

 

 

 

 

Synovial sarcoma â Epithelial cells in a gland-like pattern and spindle cells Fibrosarcoma â Fascicles of spindle cells in a herringbone pattern Epithelioid sarcoma â Nodules of epithelial cells with central necrosis Rhabdomyosarcoma â Small blue cells in an alveolar pattern

 

Correct Answer: Spindle cells in short fascicles in a storiform pattern around slit-like vessels

 

 

3721. (1346) Q9-1716:

Synovial sarcomas occur in close association to all of the following locations except:

 

1) Tendon sheaths

3) Joint capsules

2) Bursae

5) Ligaments and fascial structures

4) Within synovial joints

 

Synovial sarcomas rarely occur within a joint cavity (less than 5% of cases). Synovial sarcomas are closely associated to:

 

 

Tendon sheaths Bursae

 

Joint capsules

 

 

Ligaments, fascial structures, and aponeuroses Parapharyngeal region

 

 

Abdominal wall Pleura

 

Heart

Correct Answer: Within synovial joints

 

 

3722. (1347) Q9-1717:

Which of the following tumors may have multiple small, spotty radiopacities (focal calcification):

 

1) Malignant fibrous histiocytoma

3) Fibrosarcoma

2) Liposarcoma

5) Epitheloid sarcoma

4) Synovial sarcoma

 

Synovial sarcomas may have multiple small, spotty radiopacities caused by focal calcification (less often bone) in 15% to 20% of cases.Correct Answer: Synovial sarcoma

 

Which of the following describes the histologic appearance of a synovial sarcoma:

 

1) Storiform pattern of pleomorphic spindle cells and histiocytes

3) Spindle cells in a herringbone pattern

2) Epithelial cells and fibroblast-like spindle cells

5) Small blue cells in an alveolar pattern

4) Nodules of epithelial cells with central necrosis

 

Synovial sarcoma commonly has a bimorphic pattern with epithelial cells and fibroblast-like spindle cells. The histological characteristics include:

 

Epithelial cells: Large, round or oval, vesicular nuclei and abundant pale staining cytoplasm. The cells are in solid cords, nests, and glandular structures or cyst-like structures.

 

Spindle cells: Well-oriented, rather plump, spindle cell-shaped cells with small amounts of indistinct cytoplasm and oval dark staining nuclei. Cells form solid compact sheaths.

Correct Answer: Epithelial cells and fibroblast-like spindle cells

 

 

3724. (1349) Q9-1719:

Which of the following cytogenetic findings is common in synovial sarcoma:

 

1) Balanced reciprocal translocation (t[X;18][p11.2;q11.2])

3) Reciprocal transformation between chromosome 12 and 16

2) Ring and giant chromosomes

5) Translocation between 11 and 22

4) Translocation between chromosomes 9 and 22

 

Approximately 90% of synovial sarcomas have a balanced reciprocal translocation (t[X;18][p11.2;q11.2]) that occurs between chromosomes X and 18.

Many benign and malignant soft tissue tumors have specific chromosomal abnormalities:

 

Well-differentiated liposarcoma Myxoid liposarcoma

 

Ewings/primitive neuroectodermal tumors

Giant marker and ring chromosomes Translocation between chromosomes 12 and 16

Translocation between chromosome 11 and 22

Synovial sarcoma Translocation between chromosomes X and 18

Myxoid chondrosarcoma Translocation between

chromosomes 9 and 22

Correct Answer: Balanced reciprocal translocation (t[X;18][p11.2;q11.2])

 

Which of the following is an unfavorable prognostic factor in patients with synovial sarcoma:

 

1) Age younger than 25 years

3) Absence of poorly differentiated zones

2) Extensively calcified lesions

5) Distal extremity location

4) Size larger than 5 cm

 

Favorable prognostic factors in synovial sarcoma include: Size smaller than 5 cm

 

Age younger than 25 years

 

 

Absence of poorly differentiated areas Extensively calcified lesions

 

Distal extremity location Unfavorable prognostic factors include:

 

 

Size larger than 5 cm Age older than 40 years

 

 

Poorly differentiated areas Correct Answer: Size larger than 5 cm

 

3726. (1351) Q9-1721:

Which of the following is a favorable prognostic sign in patients with a synovial sarcoma:

 

1) Size larger than 5 cm

3) Extensively sclerotic lesion

2) Proximal location

5) Age older than 40 years

4) Extensive tumor necrosis

 

Favorable prognostic factors in synovial sarcoma include: Size smaller than 5 cm

 

Age younger than 25 years

 

 

Absence of poorly differentiated areas Extensively calcified lesions

 

Distal extremity location Unfavorable prognostic factors include:

 

 

Size larger than 5 cm Age older than 40 years

 

Poorly differentiated areas

 

Histologic features associated with a poor prognosis include: Presence of rhabdoid cells

 

Extensive tumor necrosis

 

 

High mitotic index (greater than 10 mitoses per high power field) High nuclear grade

Correct Answer: Extensively sclerotic lesion

 

 

 

 

Slide 1 Slide 2

A 12-year-old boy has a 2-week history of increasing knee pain. He is active in cross-country running. Anteroposterior and oblique radiographs are shown in Slides 1 and 2. The most likely diagnosis is:

 

1) Giant cell tumor

3) Osteochondroma

2) Chondroblastoma

5) Periosteal chondroma

4) Nonossifying fibroma

 

This lesion is a nonossifying fibroma and has a characteristic appearance which includes the following radiographic features: Eccentric

 

 

Metaphyseal Sclerotic rim

 

 

Overlying cortex is thinned Lesion is based on the cortex

 

 

 

 

 

Correct Answer: Nonossifying fibroma 3728. (1353) Q9-1723:

 

Slide 1 Slide 2

A 12-year-old boy has a 2-week history of increasing knee pain. He is active in cross-country running. Anteroposterior and oblique radiographs are shown in Slides 1 and 2. The most likely cause of the knee pain is:

 

1) Tumor destruction of the bone

3) Stress fracture

2) Pressure within the medullary cavity

5) Growing pains

4) Physeal separation

 

There is a stress fracture present on the lateral superior border of the lesion. Notice the condensation of bone in this area. This lesion is a nonossifying fibroma and has a characteristic appearance which includes the following radiographic features:

 

Nonossifying fibromas have the following radiographic features: Eccentric

 

 

Metaphyseal Sclerotic rim

 

 

Overlying cortex is thinned Lesion is based on the cortex

Correct Answer: Stress fracture

 

 

Slide 1

Which of the following conditions is rarely associated with the lesion shown in Slide 1:

 

1) Primary hyperparathyroidism

3) Polycystic kidneys

2) Hypophosphatemic vitamin D refractory rickets and osteomalacia

5) Atlantoaxial instability

4) Clubfoot and vertical talus

 

Nonossifying fibroma is a common benign bone tumor that is rarely associated with hypophosphatemic vitamin D refractory rickets and osteomalacia. Once the nonossifying fibroma is removed, the rickets resolve.

 

Other tumors that may be associated with hypophosphatemic vitamin D refractory rickets and osteomalacia include: Giant cell tumor

 

 

 

Osteoblastoma Fibrous dysplasia Hemangiopericytoma

 

 

 

 

 

 

Correct Answer: Hypophosphatemic vitamin D refractory rickets and osteomalacia 3730. (1355) Q9-1725:

 

Slide 1 Slide 2

A 12-year-old boy has a 2-week history of increasing knee pain. He is active in cross-country running. Anteroposterior and oblique radiographs are shown in Slides 1 and 2. Which of the following is the best form of treatment:

 

1) Wide resection and reconstruction

3) Chemotherapy and external beam irradiation

2) Preoperative chemotherapy, wide resection, and reconstruction

5) Curettage and bone grafting

4) Above the knee disarticulation

 

The diagnosis in this case can be made without a biopsy based upon the radiographs. Nonossifying fibromas have the following radiographic features:

 

 

 

Eccentric Metaphyseal Sclerotic rim

 

 

Overlying cortex is thinned Lesion is based on the cortex

Correct Answer: Curettage and bone grafting

 

 

 

 

 

 

Slide 1 Slide 2

A 55-year-old man presents with a 4-month history of shoulder discomfort. The plain radiographs are shown in Slide 1 and a biopsy in Slide 2. The most likely diagnosis is:

 

1) Metastatic bone disease

3) Multiple myeloma

2) Lymphoma

5) Malignant fibrous histiocytoma

4) Chondrosarcoma

 

The plain radiograph of the shoulder shows lytic lesions in the scapular spine, distal clavicle, and the proximal third of the clavicle. There is an elliptical erosion in the proximal clavicle. These three lesions suggest a diagnosis of metastatic bone disease or multiple myeloma. The biopsy specimen shows plasma cells. The plasma cells have these characteristic features:

 

 

Eccentrically placed nucleus

 

 

Peripheral clumping of the nuclear chromatin A perinuclear halo

 

The most likely diagnosis for this patient is multiple myeloma. Correct Answer: Multiple myeloma

 

3732. (1357) Q9-1727:

 

 

 

Slide 1 Slide 2

A 55-year old man presents with a 4-month history of shoulder discomfort. The plain radiographs are shown in Slide 1 and a biopsy in Slide 2. The most appropriate treatment would be:

 

1) Resection of the clavicle and scapula spine en bloc

3) Systemic chemotherapy

2) Curettage and bone grafting

5) Subacromial cortisone injection

4) Forequarter amputation

 

The plain radiograph of the shoulder shows lytic lesions in the scapular spine, distal clavicle, and the proximal third of the clavicle. There is an elliptical erosion in the proximal clavicle. These three lesions suggest a diagnosis of metastatic bone disease or multiple myeloma. The biopsy specimen shows plasma cells. The plasma cells have these characteristic features:

 

 

Eccentrically placed nucleus

 

 

Peripheral clumping of the nuclear chromatin A perinuclear halo

The diagnosis is multiple myeloma. The treatment of multiple myeloma is systemic chemotherapy (usually prednisone and an alkylating agent). External beam radiation is used to control pain and stop progressing lesions. When there is no risk of impending fracture, chemotherapy treatment is used first.

 

Correct Answer: Systemic chemotherapy

 

 

 

 

 

 

Slide 1 Slide 2

A 55-year-old man presents with a 4-month history of shoulder discomfort. The plain radiographs are shown in Slide 1 and a biopsy in Slide 2. Which of the following tests is recommended to assess the extent of this condition (staging studies):

 

1) Computerized tomography scan and technetium bone scan

3) Skeletal survey and bone marrow aspiration

2) Technetium bone scan alone

5) Positron emision tomography scan of the brain and computerized tomography of the chest

4) Computerized tomography of the chest and abdomen and technetium bone scan

 

The plain radiograph of the shoulder shows lytic lesions in the scapular spine, distal clavicle, and the proximal third of the clavicle. There is an elliptical erosion in the proximal clavicle. These three lesions suggest a diagnosis of metastatic bone disease or multiple myeloma. The biopsy specimen shows plasma cells. The plasma cells have these characteristic features:

 

 

Eccentrically placed nucleus

 

 

Peripheral clumping of the nuclear chromatin A perinuclear halo

The diagnosis is multiple myeloma. Staging is important and is performed with a skeletal survey, bone marrow biopsy, and chemistry studies including kidney function and serum calcium determination.

Technetium bone scans have a high false-negative rate and are not used in the staging process. Computerized tomography scans and brain scans are not routinely used.

Correct Answer: Skeletal survey and bone marrow aspiration

 

 

 

Slide 1 Slide 2

A 55-year-old man presents with a 4-month history of shoulder discomfort. The plain radiographs are shown in Slide 1 and a biopsy in Slide 2. Which of the following tests is not recommended to assess the extent of this condition (staging studies):

 

1) Bone marrow biopsy

3) Serum creatinine and blood urea nitrogen

2) Serum calcium level

5) Skeletal survey

4) Technetium bone scan

 

The plain radiograph of the shoulder shows lytic lesions in the scapular spine, distal clavicle, and the proximal third of the clavicle. There is an elliptical erosion in the proximal clavicle. These three lesions suggest a diagnosis of metastatic bone disease or multiple myeloma. The biopsy specimen shows plasma cells. The plasma cells have these characteristic features:

 

 

Eccentrically placed nucleus

 

 

Peripheral clumping of the nuclear chromatin A perinuclear halo

The diagnosis is multiple myeloma. Staging is important and is performed with a skeletal survey, bone marrow biopsy, and chemistry studies including kidney function and serum calcium determination.

Technetium bone scans have a high false-negative rate and are not used in the staging process. Computerized tomography scans and brain scans are not routinely used.

Correct Answer: Technetium bone scan

 

 

 

Slide 1 Slide 2

A 55-year old man presents with a 4-month history of shoulder discomfort. The plain radiographs are shown in Slide 1 and a biopsy in Slide 2. Which of the following tests would be ordered to further confirm the diagnosis:

 

1) Serum protein electrophoresis

3) Urine hydroxyproline levels

2) Serum alkaline phosphatase level

5) Serum prostate specific antigen level

4) Serum parathyroid hormone level

 

The plain radiograph of the shoulder shows lytic lesions in the scapular spine, distal clavicle, and the proximal third of the clavicle. There is an elliptical erosion in the proximal clavicle. These three lesions suggest a diagnosis of metastatic bone disease or multiple myeloma. The biopsy specimen shows plasma cells. The plasma cells have these characteristic features:

 

 

Eccentrically placed nucleus

 

 

Peripheral clumping of the nuclear chromatin A perinuclear halo

The diagnosis is multiple myeloma. Staging is important and is performed with a skeletal survey, bone marrow biopsy, hemoglobin level, and chemistry studies including kidney function and serum calcium determination.

Serum protein electrophoresis is performed to assess the amount of abnormal gammaglobulin in the serum. There are three major criteria in the diagnosis of multiple myeloma:

  1. Plasmacytoma on tissue biopsy

  2. Bone marrow plasmacytosis greater than 30% plasma cells

  3. Monoclonal immunoglobulin spike on serum protein electrophoresis exceeding 3.5 g/dl for G peaks or 2.0 g/dl for A peaks or 1.0 g/24 hours of kappa or lamba light chain excretion on urine electrophoresis

Correct Answer: Serum protein electrophoresis

 

 

 

Slide 1 Slide 2

A 55-year old man presents with a 4-month history of shoulder discomfort. The plain radiographs are shown in Slide 1 and a biopsy in Slide 2. Which of the following tests will probably be abnormal:

 

1) Serum alkaline phosphatase levels

3) Serum parathyroid hormone level

2) Urine hydroxyproline levels

5) Serum prostate specific antigen level

4) Serum hemoglobin and erythrocyte sedimentation rate

 

The plain radiograph of the shoulder shows lytic lesions in the scapular spine, distal clavicle, and the proximal third of the clavicle. There is an elliptical erosion in the proximal clavicle. These three lesions suggest a diagnosis of metastatic bone disease or multiple myeloma. The biopsy specimen shows plasma cells. The plasma cells have these characteristic features:

 

 

Eccentrically placed nucleus

 

 

Peripheral clumping of the nuclear chromatin A perinuclear halo

The diagnosis is multiple myeloma. Staging is important and is performed with a skeletal survey, bone marrow biopsy, hemoglobin level, and chemistry studies including kidney function and serum calcium determination.

 

The serum hemoglobin and erythrocyte sedimentation rates will be abnormal in approximately two-thirds of patients. Correct Answer: Serum hemoglobin and erythrocyte sedimentation rate